Центробежная сила автомобиля: Величина центробежной силы с увеличением скорости движения на повороте

Содержание

Величина центробежной силы с увеличением скорости движения на повороте

Главная » Разное » Величина центробежной силы с увеличением скорости движения на повороте

Тема 26.5. Что такое занос автомобиля, и как с ним бороться. — Автошколадома

На сухой дороге колёса надёжно держатся за дорожное покрытие, и центробежная сила не может снести автомобиль.

И вот, что ещё важно знать водителю. Самое низкое расположение центра тяжести – у пустого автомобиля. При полной нагрузке (с грузом в багажнике и пассажирами в салоне) расположение центра тяжести существенно увеличивается.

А центробежная сила как раз и приложена к центру тяжести автомобиля, и при прохождении поворота это необходимо учитывать.

С грузом и пассажирами вероятность опрокинуться выше!

Задача 34

Легковой автомобиль более устойчив против опрокидывания на повороте:

1. Без пассажиров и груза.

2. Без пассажиров, но с грузом на верхнем багажнике.

3. С пассажирами, но без груза.

4. С пассажирами и грузом.

А теперь вспоминаем курс школьной физики:

 

Центробежная сила прямопропорциональна массе автомобиля, прямопропорциональна квадрату скорости и обратно пропорциональна радиусу поворота.

Если скорость увеличить , центробежная сила увеличится

И наоборот, если скорость уменьшить ,  центробежная сила станет меньше

Задача 35

С увеличением скорости движения на повороте величина центробежной силы:

1. Не изменяется.

2. Увеличивается пропорционально скорости.

3. Увеличивается пропорционально квадрату скорости.

Задача 36

Какие действия водителя приведут к уменьшению центробежной силы, возникающей на повороте?

1. Увеличение скорости движения.

2. Снижение скорости движения.

3. Уменьшение радиуса прохождения поворота.

Что интересно! Даже не зная о существовании этой формулы, в жизни мы поступаем строго в соответствии с ней – перед входом в поворот снижаем скорость, а, проходя поворот, стараемся по максимуму «спрямить кривую», то есть по возможности стараемся увеличить радиус поворота.
Такие действия подсказывает нам вестибулярный аппарат, заложенный в нас Создателем.

А что будет, если в процесс прохождения поворота нажать на педаль тормоза?

При любом торможении вес автомобиля переносится на передние колеса. То есть передние колёса крепко прижимаются к дороге, а задние колёса наоборот стремятся оторваться от дороги.

В такой ситуации достаточно небольшого бокового усилия, чтобы задняя ось автомобиля начала вращаться вокруг передней оси.

Это явление называют

Откуда возьмется это боковое усилие? К величайшему сожалению оно обязательно возьмется, и причин для этого предостаточно. Чего стоит только одна центробежная сила!

При прохождении любого поворота на автомобиль обязательно действует центробежная сила, приложенная к центру тяжести машины.

Поскольку передние колёса всегда лучше держат дорогу (они нагружены тяжёлым двигателем), то, как правило, центробежная сила сдвигает в сторону заднюю ось. Происходит занос автомобиля при прохождении поворота.

Если сейчас со страху тормозить, к центробежной силе добавятся ещё две – тормозящее усилие передних колёс, и сразу же возникающая сила инерции.

Глядя на рисунок, должно быть понятно, что сейчас машину выбросит на обочину и там она обязательно перевернётся.

Поэтому тормозить в процессе поворота крайне нежелательно. Снижать скорость нужно до входа в поворот, а сам поворот следует проходить, что называется, «в натяжку».

То есть на педаль газа давим, но очень несильно так, чтобы автомобиль проходил поворот и без замедления, и без ускорения. В этом случае никакие силы (кроме центробежной) на автомобиль не действуют, а саму центробежную силу мы уменьшили до безопасного предела, снизив скорость до входа в поворот.

Задача 37

Что следует сделать водителю, чтобы предотвратить возникновение заноса при проезде крутого поворота?

1. Перед поворотом снизить скорость и выжать педаль сцепления, чтобы дать возможность автомобилю двигаться накатом на повороте.

2. Перед поворотом снизить скорость, при необходимости включить пониженную передачу, а при проезде поворота не увеличивать резко скорость и не тормозить.

3. Допускается любое из перечисленных действий.

Необходимо понимать – для того чтобы создались условия для заноса автомобиля,

вовсе не обязательно двигаться по криволинейному участку дороги.

Занос автомобиля может произойти и на прямолинейном участке, и порой для этого достаточно просто тормознуть или, наоборот, резко нажать на педаль газа, или резко вывернуть рулевое колесо при объезде препятствия.

Ответ очень простой – надо немедленно избавиться от причины, вызвавшей занос!

При торможении автомобиль тащит вперёд одна единственная сила – сила инерции. И приложена эта сила к центру тяжести автомобиля.

А сопротивляются силе инерции целых четыре силы, а именно, тормозящие усилия четырёх колёс автомобиля. При этом основная нагрузка ложится на тормозные механизмы передних колёс  (не зря передние тормозные колодки изнашиваются быстрее задних).

Итак, при торможении задние колёса слабо прижаты к дороге и потому склонны к блокировке. Достаточно резко нажать на педаль тормоза, и вот они уже не катятся, а скользят, потеряв сцепление с дорожным покрытием. В этом случае практически всё торможение осуществляется только передними колёсами.

А теперь представим, что левое переднее колесо тормозит эффективнее правого. Этому может быть множество причин – например, различное давление в шинах, или слева асфальт сухой, а справа влажный. Да порой достаточно, чтобы одно из колёс катилось по дорожной разметке, а другое по асфальту!

В этом случае при торможении сразу же возникает момент сил, стремящихся развернуть автомобиль.

В результате левая часть автомобиля начинает двигаться медленнее, чем правая. Происходит занос задней оси автомобиля или просто занос автомобиля.

Если сейчас не прекратить торможение, дальнейшее движение будет напоминать движение камня, брошенного на лёд – камень крутится-вертится, но летит по прямой туда, куда его тащит сила инерции.

Первая естественная реакция неопытного водителя – давить на тормоз ещё сильнее. Как вы понимаете, это означает, что занос будет продолжаться.

Изменить ситуации может обратное действие – убрать ногу с педали тормоза.

Убрали ногу с педали тормоза, и сразу же исчез момент сил, разворачивавших автомобиль (колёса свободно катятся). Но сила инерции никуда не делась, она по-прежнему тащит автомобиль вперёд!

Не беда, поворачиваем рулевое колесо в сторону заноса и выравниваем траекторию движения автомобиля.

(Сравните этот рисунок с верхним. Видите, как на этом рисунке водитель повернул передние колёса в сторону заноса).

Примечание. Как мы уже определились, занос автомобиля – это занос именно задней оси. Задние колеса стремятся сблизиться с передними. В этом случае, выравнивая автомобиль, водитель поворачивает рулевое колесо навстречу приближающимся задним колёсам.

Это и принято называть «поворот рулевого колеса ».

Задача 38

Для предотвращения заноса, вызванного торможением, водитель в первую очередь должен:

1. Прекратить начатое торможение.

2. Выключить сцепление.

3. Продолжить торможение, не изменяя усилия на педаль.

При разгоне расклад сил прямо противоположный.

Теперь сила инерции направлена назад, а вперёд автомобиль тянут ведущие колёса. И если ведущие колёса надёжно держат дорогу (не буксуют), то и автомобиль ведёт себя идеально, послушно выполняя все желания водителя.

Однако нет никакой гарантии, что левые и правые колёса всегда держатся за дорогу абсолютно одинаково. Мы уже упоминали о возможной разнице давления в шинах, или, скажем, слева проезжая часть сухая, а справа влажная.

Поэтому занос можно получить не только при торможении, но и при ускорении.

Достаточно резко нажать на педаль газа (особенно на скользком покрытии) и ведущие колёса начнут вращаться с пробуксовкой. А любое проскальзывание колёс – это потеря сцепления с дорогой.

Так что во всех случаях рецепт один – ,

то есть в данном случае – уменьшить нажатие на педаль управления подачей топлива.

Задача 39

Как водитель должен воздействовать на педаль управления подачей топлива при возникновении заноса, вызванного резким ускорением движения?

1. Усилить нажатие на педаль.

2. Не менять положения педали.

3. Ослабить нажатие на педаль.

Иногда водителям приходится резко вильнуть при объезде препятствия.

Представим, что водитель, двигаясь со скоростью 60 км/ч, в последний момент решил объехать канализационный люк.

Но ведь резкий поворот направляющих колёс это тоже своеобразное торможение. В прямом направлении скорость автомобиля резко падает, и машина заметно приседает на передние колёса.

А раз есть торможение, сразу же появляется сила инерции, при этом корпус автомобиля уже развёрнут – идеальные условия для заноса!

Летом на сухом асфальте ничего страшного не случится, просто машину качнёт туда-сюда при объезде препятствия.

Но зимой на скользкой дороге занос гарантирован. Более того – в следующее мгновение скользить будут все четыре колеса.

Да и летом, если скорость под сотню, события будут развиваться точно так же.

Что делать?

Да всё то же самое. Как только водитель почувствовал, что автомобиль уходит в занос, надо немедленно избавиться от причины, вызвавшей занос. И теперь уже бог с ним, с этим люком.

Быстро (но плавно!) поворачиваем рулевое колесо в сторону заноса.

Передние колёса «цепляют» дорогу (перестают скользить), управляемость автомобиля восстанавливается, и машина послушно возвращается на свою полосу.

Задача 40

Что следует предпринять водителю для предотвращения опасных последствий заноса автомобиля при резком повороте рулевого колеса на скользкой дороге?

1. Быстро, но плавно повернуть рулевое колесо в сторону заноса, затем опережающим воздействием на рулевое колесо выровнять траекторию движения автомобиля.

2. Выключить сцепление и повернуть рулевое колесо в сторону заноса.

3. Нажать на педаль тормоза и воздействием на рулевое колесо выровнять траекторию движения.

Пришло время поговорить о различии в управлении автомобилем и

И тот, и другой совершенно одинаково уходят в занос. Но вот выбираются из заноса по-разному. Связано это с тем, что задние колёса автомобиль, а передние – автомобиль.

Представьте человека, который привязал к спинке санок палку и пытается ею толкать эти санки.

Ведь они тут же начнут складываться влево или вправо. То есть по аналогии с автомобилем заднюю ось будет заносить толкающее усилие.

Если же человек догадается привязать палку или просто веревку спереди и потянет санки, то они будут следовать за ним, как нитка за иголкой без всяких заносов.

Именно этим и отличается передний привод от заднего. Если задние колёса толкают массу, расположенную перед ними, то передние колёса тянут массу, расположенную после них.

Именно поэтому, выходя из заноса на заднем приводе, мы плавно уменьшаем нажатие на педаль газа, пытаясь усмирить центробежную силу и восстановить управляемость автомобиля.

И именно поэтому на переднем приводе, мы слегка увеличиваем нажатие на педаль газа, чтобы передние колёса вытащили нас из заноса.

Итак, на повороте возник занос задней оси автомобиля (задние колёса скользят по дороге, и центробежная сила несёт их на обочину). И именно задние колёса у нас ведущие.

Если сейчас добавить крутящий момент на ведущие колёса (то есть нажать на педаль газа) ситуация только усугубится – мало того, что задние колёса скользят, так теперь ещё и буксуют, и сцепление с дорогой потеряно окончательно.

В то же время нельзя и нажимать на педаль тормоза или резко бросать газ – в этом случае к центробежной силе добавиться ещё и сила инерции, и это только усилит занос.

Вспоминаем наш общий универсальный принцип – надо избавиться от причины, вызвавшей занос.

А заносит нас центробежная сила. Ну, совсем-то от неё избавиться невозможно, но можно её уменьшить, если снизить скорость.

Только снижать скорость нужно плавно, слегка уменьшая подачу топлива, одновременно поворачивая рулевое колесо в сторону заноса.

После того как управляемость автомобиля восстановится, завершаем поворот.

Задача 41

Что следует предпринять, если на  повороте возник занос задней оси заднеприводного автомобиля?

1. Увеличить подачу топлива, рулевым колесом стабилизировать движение.

2. Притормозить и повернуть рулевое колесо в сторону заноса.

3. Значительно уменьшить подачу топлива, не меняя положения рулевого колеса.

4. Слегка уменьшить подачу топлива и повернуть рулевое колесо в сторону заноса.

И опять на повороте возник занос задней оси автомобиля. Только на этот раз автомобиль переднеприводной.

Как вы думаете, если сейчас повернуть рулевое колесо в сторону заноса и добавить крутящий момент на ведущие колёса, вытянут ли передние колёса нас из заноса?

Только помните!

Наращивать давление на педаль газа нужно слегка, очень плавно и очень осторожно, не допуская пробуксовки передних колёс. Как же они будут тянуть, если начнут буксовать?

Задача 42

На повороте возник занос задней оси переднеприводного автомобиля. Ваши действия?

1. Уменьшите подачу топлива, рулевым колесом стабилизируете движение.

2. Притормозите и повернёте рулевое колесо в сторону заноса.

3. Слегка увеличите подачу топлива, корректирую направление движения рулевым колесом.

4. Значительно  увеличите подачу топлива, не меняя положение рулевого колеса.

Задача 43

Устранение заноса задней оси путём увеличение скорости возможно:

1. Только на переднеприводном автомобиле.

2. Только на заднеприводном автомобиле.

3. На любом автомобиле из перечисленных.

Современные автомобили снабжены всевозможными устройствами, помогающими водителю избегать неприятностей на дороге.

К числу таких умных устройств, прежде всего, относится – антиблокировочная тормозная система.

Однако следует знать, что антиблокировочная система очень хороша только на прямолинейных участках. При торможении она так умело перераспределяет тормозное усилие по колёсам автомобиля, что все четыре колеса всегда надёжно держатся за дорогу. А это, в свою очередь, исключает занос автомобиля.

Но против бокового усилия, то есть против центробежной силы, возникающей на повороте, АБС бессильна.

На сухом покрытии центробежная сила может попросту опрокинуть автомобиль.

На скользком покрытии та же центробежная сила может легко занести заднюю ось автомобиля…

… или даже полностью снести автомобиль с дороги. И тут никакая АБС не поможет.

Задача 44

Исключает ли антиблокировочная тормозная система (АБС) возможность возникновения заноса или сноса при прохождении поворота?

1. Полностью исключает возникновение только заноса.

2. Полностью исключает возникновение только сноса.

3. Не исключает возможность возникновения сноса или заноса.

Учебный предмет «Основы управления транспортными средствами» / КонсультантПлюс

Задача 1.

Двигаясь в прямом направлении, Вы попали на небольшой участок обледенелой дороги. Что следует предпринять в такой ситуации?

1. Не меняя положения рулевого колеса и скорости движения, проехать скользкий участок дороги.

2. Не меняя положения рулевого колеса, выключить передачу и двигаться накатом.

3. Не меняя положения рулевого колеса, увеличить скорость на этом участке.

Задача 2.

Как влияет увеличение скорости движения на величину центробежной силы при повороте?

1. Центробежная сила увеличивается.

2. Центробежная сила не изменяется.

3. Центробежная сила уменьшается.

Задача 3.

Как должен действовать водитель, если произошел внезапный разрыв шины переднего колеса автомобиля?

1. Пытаться сохранить прямолинейное движение и резко затормозить.

2. Пытаться сохранить прямолинейное движение и плавно затормозить до полной остановки автомобиля.

Задача 4.

Какое транспортное средство, движущееся во встречном направлении, создает иллюзию, что оно движется с большей скоростью, чем в действительности?

1. Транспортное средство, имеющее большие габариты (автопоезд, автобус).

2. Мотоцикл.

3. Легковой автомобиль.

Задача 5.

В каком случае при движении на повороте дороги устойчивость автомобиля будет выше?

1. При движении с большей скоростью.

2. При движении с меньшей скоростью.

Задача 6.

Какие последствия может вызвать размещение тяжелого груза на багажнике, установленном на крыше легкового автомобиля?

1. Уменьшит устойчивость автомобиля против опрокидывания.

2. Увеличит устойчивость автомобиля против опрокидывания.

3. Уменьшит длину тормозного пути автомобиля.

Задача 7.

В каком случае создается иллюзия, что скорость автомобиля меньше, чем в действительности?

1. При движении по дороге, проходящей на открытой местности.

2. При движении по лесной дороге.

Задача 8.

Может ли произойти боковой занос автомобиля, оборудованного антиблокировочной системой тормозов (ABS) при движении на закруглении дороги?

1. Может.

2. Не может.

Задача 9.

Что рекомендуется водителю при движении по дороге, покрытой грязью?

1. Увеличить интервал и дистанцию, снизить скорость движения.

2. Уменьшить интервал и дистанцию.

3. Снизить скорость движения.

Задача 10.

Влияет ли на устойчивость автомобиля величина радиуса поворота дороги?

1. Влияет.

2. Не влияет.

Правильные ответы

Открыть полный текст документа

Центробежная сила и смещение на повороте

При движении на любом повороте возникает центробежная сила, стремящаяся занести автомобиль или опрокинуть его (см.рисунок 1).  m – масса автомобиля; V – скорость движения автомобиля; R – радиус поворота автомобиля

Рисунок 1 – Возникновение центробежной силы на повороте

Как видно из рисунка величина центробежной силы, Fтр, увеличивается пропорционально квадрату скорости, V, поэтому на повороте в первую очередь нужно снижать скорость. Выполнять поворот необходимо таким образом, чтобы траектория движения снижала вероятность смещения  автомобиля на полосу, предназначенную для встречного движения под действием центробежной силы. Поэтому необходимо начинать поворот с внешней границы полосы движения (чем больше радиус прохождения поворота, тем меньше центробежная сила).  На величину центробежной силы влияет и масса транспортного средства, поэтому наиболее устойчив автомобиль против опрокидывания без груза и пассажиров (с меньшей массой). При меньшей массе центр тяжести автомобиля будет ниже, поэтому будет и меньше опрокидывающий момент от центробежной силы. Водителям необходимо помнить, что при маневрировании (при поворотах, разворотах, перестроениях) передние и задние колеса (тем более колеса прицепа автопоезда относительно тягача или автобуса с гармошкой) имеют разные траектории движения. Смещение будет тем сильнее, чем дальше задние колёса автомобиля (колеса прицепа или гармошки автобуса) от передних колес (рисунок 2). Смещение всегда будет происходить к центру поворота.  

Рисунок 2 – Смещение колес на повороте

 

   

Автошкола «Профессионал» желает безаварийной езды. Успехов в изучении ПДД.   

 

Хотите записаться на обучение в автошколу, но у вас остались вопросы? Позвоните нам по телефону +7 (902) 446-17-35 Или приходите в офис по адресам ул. Техническая 32, ул. Сибирский тракт 8Д, офис 210

17.05.2017

центробежная, пдд, автошкола

« Предыдущая страница Следующая страница »

Навигация

Стоимость обучения

Контакты

  • Автошкола на Сортировке (Техническая 32)
  • Автошкола на Шарташском рынке

Обучение в автошколе

Наше сообщество

Полезные ресурсы

Последние добавленные

Новости автошколы

Грядущие изменения в ПДД

Лицензия

Последние фото

Центробежная сила на повороте

Величина центробежной силы на повороте увеличивается пропорционально квадрату скорости,(Билет37,в19)и снижение скорости приведет к уменьшению этой силы.(Билет33,в19) Центробежная сила, наклоняющая автомобиль на повороте, условно приложена в его центре тяжести. Более устойчив автомобиль на повороте без груза и пассажиров, так как в этом случае у него самое низкое расположение центра тяжести.(Билет8,в19) На повороте из-за действия центробежной силы (всегда направлена к наружному закруглению дороги) автомобиль может смещаться. При левом повороте возможно его сползание в кювет, при правом — выезд на соседнюю или встречную полосу. С учетом вышеизложенного выбирайте безопасные траектории прохождения поворота.

При правом повороте направляйте автомобиль от левого края полосы к правому (Билет16,в19). При левом повороте — от правого края полосы к левому. (Билет32,в19).

Скорость, обгон, видимость, свет. При повороте прицеп автопоезда смещается к центру поворота (к внутреннему закруглению дороги). Учитывайте эту особенность при взаимном маневрировании с автопоездами.(Билет35,в19) С увеличением скорости движения поле зрения сужается, так как водитель в такой ситуации вынужден переводить взгляд дальше от автомобиля, чтобы контролировать ситуацию впереди на большем расстоянии.

В темное время суток и в пасмурную погоду скорость встречного автомобиля воспринимается ниже, чем в действительности, что является опасным. Также имейте в виду, что скорость крупногабаритного транспорта водители склонны переоценивать, а небольших автомобилей и мотоциклов — недооценивать. (Билет17,в20).Расстояние до предметов в условиях тумана представляется большим, чем в действительности (Билет5,в20).

Не следует вплотную двигаться за транспортным средством, которое Вы хотите обогнать. В этом случае обзор дороги сужен, и из такого положения приступать к обгону нельзя.(Билет34,в19) Значительное влияние на безопасность движения оказывает эмоциональное состояние водителя. Оно во многих случаях определяет правильность и точность действий. Известно, что радостные переживания делают человека бодрым и уверенным. В результате его действия оказываются более точными, быстрыми и координированными. Неприятности, тяжелые переживания приводят к противоположным результатам. Особенно это отражается на внимании: человек, поглощенный своими переживаниями, становится более рассеянным.

Типичными признаками наступившего утомления являются сонливость, вялость, притупление внимания. В этом случае водитель должен прекратить движение и отдохнуть, иначе он может заснуть за рулем.(Билет 6,в20)

Компенсировать недостаточно быструю реакцию Вы сможете, заранее прогнозируя развитие дорожных ситуаций.

Водитель должен вести транспортное средство с учетом видимости в направлении движения (пункт 10.1 Правил). При движении в условиях плохой видимости следует выбирать скорость исходя из того, чтобы остановочный путь был меньше расстояния видимости. (Билет25,в19) (Билет35,в20) Это позволит Вам остановить автомобиль на просматриваемом в данный момент участке местности.

Если остановочный путь превышает расстояние видимости, значит, Вы едете с опасной для данных условий скоростью.

При приближении к вершине подъема в темное время суток всегда следует переключить дальний свет на ближний (Билет10,в19),чтобы не ослепить водителя встречного транспортного средства, которое в данный момент Вы можете еще не видеть.

Блики снежинок в дальнем свете фар представляют собой очень красивое зрелище, однако ослепляют водителя. Ближний свет фар ложится световым пятном на дорогу непосредственно перед автомобилем и в сочетании с противотуманными фарами обеспечит наилучшую видимость в условиях сильной метели. (Билет38,в19)

Не нашли то, что искали? Воспользуйтесь поиском:

Page 2

При потере сцепления колес с дорогой из-за образования под ними «водяного клина»,(Билет31,в19) так называемое аквапланирование, автомобиль становится неуправляемым. Бороться с этим явлением можно только снижением скорости, применяя торможение двигателем, чтобы не вызвать занос автомобиля. Действие сильного бокового ветра наиболее опасно при выезде с закрытого участка дороги на открытый,(Билет21,в19) (Билет40,в19) так как в этом случае автомобиль может неожиданно для водителя потерять курсовую устойчивость. Обычно перед такими участками устанавливают предупреждающий знак 1.27 «Боковой ветер». Увидев такой знак, будьте готовы к возможному отклонению автомобиля от заданного курса, заблаговременно снизьте скорость. Когда правые колеса автомобиля наезжают на неукрепленную и влажную обочину, а левые остаются на проезжей части (под ними больший коэффициент сцепления, чем под правыми), рекомендуется, не прибегая к торможению (чтобы не спровоцировать занос), плавно вернуть автомобиль на проезжую часть.(Билет1,в19) Двигаться по глубокому снегу следует на заранее выбранной пониженной передаче(Билет7,в19), которая исключала бы остановку автомобиля и последующее буксование колес при трогании. После проезда через водную преграду из-за попадания воды в ступицы колес значительно снижаются тормозные свойства автомобиля. Для восстановления прежней эффективности торможения следует просушить тормозные колодки, двигаясь на небольшой скорости, непродолжительным многократным нажатием на педаль тормоза.(Билет29,в19) При длительном торможении с выключенным сцеплением (передачей) на крутом спуске автомобиль под действием собственного веса разгоняется. Для поддержания постоянной скорости движения в этом случае водитель вынужден использовать рабочую тормозную систему (нажатие на педаль тормоза). При таком длительном использовании рабочей тормозной системы возможен перегрев тормозных механизмов и уменьшение эффективности торможения.(Билет23,в19) При торможении двигателем на крутом спуске выбирайте передачу, исходя из следующих условий: чем круче спуск, тем ниже передача.(Билет15,в19) Это позволит поддерживать постоянную невысокую скорость движения и уверенно управлять автомобилем. Правильное положение рук на руле — примерно в секторах от «без четверти три до без десяти два», если принять руль за циферблат часов. Такое расположение рук легко позволяет повернуть руль в любую сторону.(Билет30,в19) При трогании на подъеме стояночный тормоз следует начинать отпускать одновременно с началом движения(Билет39,в19), чтобы избежать скатывания автомобиля (при отпускании до начала движения) и остановки двигателя (при запаздывании отпускания). При парковке автомобиля на подъеме или спуске дороги с тротуаром передние колеса поверните так, чтобы автомобиль собственным весом прижимал их к тротуару.(Билет11,в19) На дороге с обочиной колеса поверните вправо по ходу движения, т. е. в сторону обочины. Тогда при несанкционированном движении автомобиль откатится на обочину, а не на проезжую часть.(Билет 6,в19)

Разгон-торможение, расход топлива, разворот с использованием прилегающих территорий Вероятность возникновения аварийной ситуации при движении в плотном транспортном потоке будет меньше, если скорость Вашего автомобиля будет равна средней скорости потока. (Билет 5,в19) При плавном разгоне и плавном торможении Вам и Вашим пассажирам будет комфортно. Кроме того, такой стиль вождения обеспечит наименьший расход топлива (Билет22,в19). Значительно увеличивают расход топлива резкие разгоны и длительное движение на пониженных передачах.(Билет14,в19) Правила запрещают движение задним ходом на перекрестках. Въезды на прилегающие территории (во дворы, на автозаправочные станции, предприятия и т. п.) не считаются перекрестками. Поэтому на двухполосных дорогах, в местах, где нет сплошной линии разметки, удобно выполнять разворот с использованием въездов на прилегающие территории.

Если прилегающая территория расположена справа по ходу движения (Билет6,в12), то, миновав въезд на нее, включите правый указатель поворота и остановитесь. Затем, убедившись в безопасности движения задним ходом, въезжайте на прилегающую территорию. При этом маневре на автомобиле должен быть включен правый указатель поворота. Затем, оказавшись лицом к дороге, включайте левый указатель поворота и, уступив транспорту, находящемуся на дороге, и пешеходам, путь движения которых Вы пересекаете, выезжайте с прилегающей территории.

Если прилегающая территория расположена слева (Билет9,в19), то, включив левый указатель поворота и пропустив встречный транспорт и пешеходов, путь движения которых Вы пересекаете, въезжайте на прилегающую территорию. Затем, убедившись в безопасности движения, включите правый указатель поворота и задним ходом выезжайте на дорогу.

Не нашли то, что искали? Воспользуйтесь поиском:



Движение на поворотах

Движение на поворотах

Автомобиль на повороте испытывает действие дополнительных внешних сил, в частности центробежной силы, которые отсутствуют при движении на прямолинейных участках дороги. Центробежная сила стремится сдвинуть автомобиль к внешней стороне от центра поворота дороги. Ее величина зависит от веса автомобиля, радиуса закругления и квадрата скорости. Вот почему при большой скорости автомобиля опасно делать резкие повороты рулевого колеса, выполняемые в минимальные промежутки времени: в этом случае радиус резко уменьшается, а центробежная сила соответственно увеличивается.

Существует справедливое мнение, что на дороге нет двух одинаковых поворотов. Каждый поворот имеет другой радиус, другой уклон, другое покрытие, другую видимость или другую окружающую обстановку. Поэтому водитель должен оценивать каждый поворот и выбирать скоростной режим, обеспечивающий безопасность движения именно на этом конкретном повороте с учетом присущих ему особенностей и неожиданностей На повороте никогда не следует смотреть на дорогу непосредственно перед автомобилем, Необходимо смотреть на конец поворота или на более дальний видимый участок дороги. Тогда водитель сможет не только правильно определить радиус поворота, но также увидит, не подъезжает ли кто с противоположной стороны, свободна ли дорога на его стороне.

Двигаясь на повороте, нельзя допускать скольжения, которое может привести к потере управления автомобилем. Если на повороте при сухом покрытии водитель услышит скрип шин, то, значит, он развил чрезмерную скорость. Нельзя срезать повороты, необходимо всегда ехать по правой стороне дороги. На повороте не следует тормозить, переключать передачи, чересчур резко нажимать на педаль управления открытием дросселя. Все это может послужить причиной возникновения опасной ситуации.Правильный поворот следует осуществлять следующим образом: перед приближением к его началу необходимо уменьшить скорость до соответствующего предела, определяемого на основании опыта; начало закругления следует проезжать, не увеличивая скорость вращения коленчатого вала двигателя, но и без торможения даже двигателем; одновременно без рывка, постепенно поворачивать рулевое колесо, ускоряя его вращение по мере увеличения кривизны поворота; примерно от половины поворота постепенно увеличивая частоту вращения коленчатого вала двигателя так, чтобы выйти из поворота со скоростью, равной по величине скорости перед началом прохождения поворота. Поворачивать рулевое колесо обратно в основное положение нужно также без рывка и постепенно.

Необходимо помнить, что из-за инерции движения автомобиля начинать выполнение поворота следует несколько раньше начала непосредственного закругления дороги, а выходить из поворота надо также соответственно раньше. Величина этого опережения определяется на основании длительного опыта. Правильное выполнение поворотов характеризуется отсутствием заноса и плавностью. Рулевое колесо необходимо держать всеми пальцами и ладонями обеих рук достаточно крепко, по. возможности, не следует перекладывать руки, а вращать колесо, не отнимая от него рук. Нельзя перекладывать руки крест-накрест. Если на повороте с очень большой кривизной, например на горных серпантинах, водителю нужно переменить положение одной руки, то необходимо, чтобы другая рука всегда плотно обхватывала обод рулевого колеса. Отнимать от рулевого колеса обе руки одновременно во всех случаях запрещается. Одной из трудных задач для водителя является быстрое и безошибочное определение величины кривизны поворота и отсюда выбор безопасной скорости движения.

Причина многих аварий!Центробежная сила. | Онлайн-Автошкола Романа Унуковск

Центробежная сила – это сила, возникающая на повороте при движении автомобиля, которая заносит его на обочины или опрокидывает.

На автомобиль действуют самые разные силы во время его движения: сила тяжести, сила инерции, сила сопротивления воздуха, центробежная сила и многие другие. Если водитель знает и понимает, как они действуют, то это поможет ему избежать негативных ситуаций:

1. Как не вылететь с дороги.
2. Как справиться с заносом и так далее.

Центробежная сила пытается отбросить автомобиль от центра поворота на обочину, и иногда у нее это выходит.

Когда этого не происходит?

Если у водителя по природе своей отличный вестибулярный аппарат, то он, скорее всего, поймет, когда скорость достигла своего пика, и когда её пора снизить, чтобы избежать негативный последствий.

Причина многих аварий!Центробежная сила.

Некоторые факты о центробежной силе:

1. Центробежная сила не имеет постоянной величины, а зависит от скорости, от массы транспортного средства и от радиуса поворота, т.е чем меньше m автомобиля, тем меньше сила тяжести, и тем меньше вероятность, что он опрокинется.

2. Если m увеличивается в 5 раз, то и центробежная сила увеличивается в 5 раз.
3. Когда радиус поворота увеличивается, то центробежная сила уменьшается.
4.Центробежная сила увеличивается в том случае, если уменьшается угол поворота.
5.Смещение будет сильнее, если расстояние между задними и передними колесами будет сильно увеличено.
6. Чем больше радиус прохождения поворота, тем меньше центробежная сила
7. Резина автомобиля обеспечивает пятно контактов в различных ситуациях. А от пятна контактов резины с дорожным покрытием зависит сцепление вашего автомобиля с дорогой.

Причина многих аварий!Центробежная сила.

Чтобы снизить центробежную силу, для начала необходимо снизить скорость во время прохождения поворота.
Не забывайте, что центробежная сила квадратично зависима от скорости движения, т.е. если вы увеличите скорость в 5 раз, то центробежная сила увеличиться в двадцать пять раз. Когда вы думаете о том, какую траекторию прохождения поворота выбрать, то учтите, пожалуйста, всевозможные варианты смешения, т.е. двигайтесь чуть ближе к центру поворота, так у вас будет в запасе немного расстояния до обочины. Так вы предотвратите опрокидывание автомобиля.

Удачи на дорогах!

Дорогой читатель, спасибо за то, что дочитал эту статью до конца! Если она понравилась тебе, то поставь палец вверх и подпишись на канал! А также читай по темам:

◄ «Ни за что не делайте это при ДТП!»

◄»Как трогаться в горку без отката на механике?»

◄»Как получить права международного образца?»

◄»Женщина за рулем.Мнение инструктора.»

Группа в VK — https://vk.com/unukovsky_school

Мой канал на YouTube — https://www.youtube.com/channel/UCLeTXroFxidSQa8NBsz1pLg?view_as=subscriber

БГАК — Учебные материалы — Д.В.Фокин — Современные автомобильные технологии — Теория — Тормозное управление

Системы управления тормозами

Система предотвращения опрокидывания (ROP)

Аббревиатура системы предотвращения опрокидывания — ROP — расшифровывается как Roll Over Prevention (иногда её также расшифровывают Roll-Over-Programm). Эта функция предназначена для своевременного реагирования на силы и моменты сил, которые могут привести к опрокидыванию или переворачиванию автомобиля. ROP также является только программным расширением системы ESC.

При движении в повороте автомобиль испытывает воздействие вращающего момента относительно продольной оси (крен кузова) (рис.5.2.99). Этот момент возникает из-за того, что центробежная сила и боковая сила сцепления колёс с дорогой приложены в разных точках (в центре масс и в точке контакта шин с дорогой, соотв.). Действие этого момента можно наглядно видеть, двигаясь в повороте за грузовым автомобилем с высоким фургоном.

Рисунок 5.2.99 – Схема крена автомобиля на повороте

Фургон будет наклоняться в сторону внешнего края поворота, а величина наклона будет зависеть от скорости автомобиля, массы и высоты фургона.

Поскольку шины располагают достаточным запасом сцепления, схему действующих сил можно представить себе как рычаг, точка опоры которого находится в месте контакта шин с дорогой.

Длина плеча рычага определяется расположением центра масс автомобиля. Чем выше располагается центр масс, тем более длинное плечо будет у рычага. Если центр тяжести будет расположен очень высоко, то уже небольшое боковое усилие сможет привести к опрокидыванию автомобиля.

Функция ROP предназначена для предотвращения таких ситуаций в самом начале их возникновения. Для своей работы она использует датчики системы ESC.

Принцип действия системы заключается в следующем.

В примере с фургоном при быстром проезде поворота возникают боковые силы, создающие вращающий момент относительно продольной оси автомобиля (рис.5.2.100).

Для стабилизации автомобиля система ROP подтормаживает переднее наружное колесо.

Практически все автомобили при резком объезде внезапно возникшего препятствия на дорожном покрытии с хорошим сцеплением подвергаются опасности опрокидывания. Этот манёвр распознаётся и, хотя автомобилю ещё не угрожает занос, в контуре переднего наружного колеса резко создаётся очень высокое тормозное давление.

Рисунок 5.2.100 – Работа системы ROP в различных дорожных ситуациях

Под действием этих боковых сил автомобиль опасно накреняется. Датчики системы ESC улавливают это движение и передают информацию о нём блоку управления ESC. В памяти функции ROP заложены характеристики, сравнение с которыми позволяет ESC распознать в поступающих от датчиков сигналах признаки приближающейся опасности опрокидывания автомобиля. Поскольку опасность опрокидывания с увеличением загрузки возрастает, на некоторых моделях предельные значения для ROP задаются как функция от рассчитанной массы автомобиля.

При распознавании такой опасности включается функция предотвращения опрокидывания ROP. Автомобиль стабилизируется уменьшением поперечного ускорения.

Система ESC подтормаживает переднее внешнее (по отношению к повороту) колесо. С помощью насоса обратной подачи и, при наличии, активного усилителя тормозов в контуре колеса быстро создаётся необходимое тормозное давление. Кроме того, уменьшается крутящий момент двигателя. В результате возникает момент сил, противостоящий направленным во внешнюю сторону боковым силам. Тем самым предотвращается боковое раскачивание автомобиля, которое может привести к его опрокидыванию.

Водитель может, при определённых условиях, заметить работу системы предотвращения опрокидывания, хотя он будет ещё не в состоянии распознать саму критическую ситуацию. Во время корректирующих действий системы ROP мигает контрольная лампа ESC.

На некоторых современных легковых автомобилях сегмента SUV система ESC имеет возможность распознавать установленный на крыше автомобиля багажник и вносить в связи с этим соответствующие коррективы в свои алгоритмы. Багажник на крыше существенно влияет на высоту центра масс, а тем самым и на динамическую ситуацию автомобиля.

В оригинальном продольном релинге на крыше (рис.5.2.101) установлен датчик Холла, связанный отдельным проводом с блоком управления ABS/ESC.

Рисунок 5.2.101 – Распознавание багажника на крыше

В поперечном релинге (также оригинальном) имеется постоянный магнит, который при установленном поперечном релинге инициирует соответствующий сигнал датчика.

Центробежные силы и их воздействие на шину


из «Основы современной технологии автомобильных шин»

На износ шин существенное влияние оказывают центробежные силы, возникающие при движении автомобиля. [c.61]
При прямолинейном движении автомобиля шина находится под действием центробежной силы, направленной от центра к радиусу колеса, которая, стремясь сорвать покрышку с обода, вызывает дополнительные напряжения в каркасе и способствует отслоению протектора. [c.61]
При уравновешенности шины (при равномерном распределении массы во всех симметрично расположенных участках), центробежная сила действует по окружности на все точки шины с одинаковой силой, колесо вращается равномерно и протектор изнашивается равномерно. При неуравновешенности шины (дисбалансе) колесо вращается неравномерно (бьет) и в местах утолщения возникают более высокие температуры, что может привести к разрушению покрышки. [c.61]
Дисбаланс шины является следствием технологических дефектов неравномерной толщины деталей покрышки, неоднородности резиновых смесей, неравномерности распределения стыков деталей по окружности покрышки, небрежности при монтаже шины на обод колеса — несовмещению вентиля камеры с самым легким местом покрышки, несбалансированностью шины с колесом и эксцентриситета колес и др. Так как дисбаланс шин и колес существенно влияет на эксплуатационные свойства шин и автомобиля, покрышки подвергаются специальным испытаниям на соответствие нормам по дисбалансу. [c.61]
При криволинейном движении шина дополнительно подвергается воздействию боковой силы, направленной в противоположную повороту машины сторону, перпендикулярно плоскости колеса. Эта сила стремится сорвать шину с обода, но в боковом направлении и тем самым увеличивает прогиб шины и повышает напряжение в бортах покрышки. В случае недостаточного сцепления шины с поверхностью дороги машина начинает скользить (боковой занос) и устойчивость автомобиля нарушается. Свойство шины сопротивляться боковому заносу увеличивает боковую устойчивость автомобиля и облегчает управление им. [c.61]
Яшунская Ф. И., Крицков В. Ф., Рагимов С. И. К вопросу об экономической эффективности шин типа Р. Автомобильная промышленность , 1970, 2, с. 31-33. [c.61]

Вернуться к основной статье

Физика и правила ДД

  1. Прохождение поворотов

Изменение направления движения автомобиля достигается поворачиванием передних колес влево или вправо при помощи рулевого управления.

Для того чтобы при движении автомобиля на повороте колеса его имели качение без скольжения, они должны катиться по окружностям, описанным из одного центра — центра поворота, лежащего в точке О, на линии задней оси автомобиля. В результате при повороте колеса автомобиля описывают окружности разного радиуса: передние колеса, описывают окружность большего радиуса, чем задние.

Безопасность движения на закруглениях пути (поворотах) определяется следующими условиями.

Когда автомобиль движется по окружности, возникает центростремительная сила, удерживающая автомобиль на окружности, и центробежная сила, стремящаяся отбросить автомобиль от центра вращения.

Величина центробежной силы непостоянна и зависит от массы автомобиля, радиуса поворота и скорости движения.

Чем больше масса автомобиля, тем больше его инерция, а следовательно, и центростремительная сила. Это, в свою очередь, вызывает увеличение центробежной силы, которая равна и противоположна центростремительной. Таким образом, между весом автомобиля (массой) и центробежной силой существует прямая пропорциональная зависимость. Это означает, что величина центробежной силы увеличивается во столько же раз, во сколько раз увеличивается масса. Если вес автомобиля возрастет в два раза, то и центробежная сила увеличится в два раза, и т. д.

Между центробежной силой и радиусом поворота зависимость обратно пропорциональная: при увеличении радиуса поворота величина центробежной силы уменьшается. И, наоборот, во сколько раз меньше угол поворота, во столько раз больше центробежная сила.

При увеличении скорости движения центробежная сила возрастает, но не в простой зависимости, а в квадратной, т. е. так же, как в зависимости от скорости движения возрастает тормозной путь автомобиля. Если скорость движения автомобиля возрастет в два раза, величина центробежной силы возрастет в четыре раза, при возрастании скорости в три раза центробежная сила увеличится в девять раз и т. д.

Опасность увеличения центробежной силы состоит в том, что, стремясь отбросить автомобиль с окружности, она может вызвать боковой занос на повороте. При всех возможных случаях бокового скольжения занос автомобиля на повороте является наиболее опасным по следующим причинам: занос на повороте, как правило, вызывает скольжение задней оси, что опаснее заноса передней оси; вывести автомобиль из заноса на повороте труднее из-за ограниченного пространства.

Рис. 5. Схема поворота автомобиля передним ходом

Занос на повороте опасен еще и тем, что чаще всего вызывает опрокидывание автомобиля, особенно при высокорасположенном центре тяжести, например при высокой укладке груза. Достаточно колесу при боковом скольжении встретить препятствие — камень, колею, канаву, как центробежная сила, приложенная к центру тяжести автомобиля, опрокинет автомобиль через это колесо.

Из сказанного должно быть понятно, что вероятность бокового заноса на повороте тем больше, чем выше скорость движения, больше вес автомобиля, меньше (круче) угол поворота, хуже сцепление колес с дорогой (когда центробежная сила может превысить центростремительную силу — силу трения).

Наиболее верным средством предупреждения опасности бокового заноса при повороте является снижение скорости движения. Прибегать к торможению на повороте ни в коем случае нельзя, так как вследствие реакции торможения уменьшается сцепление колес с поверхностью дороги, что облегчает возникновение заноса (торможение на повороте ухудшает устойчивость автомобиля).

При возникновении бокового заноса на повороте надо иметь в виду следующее: если при боковом скольжении задней оси автомобиля на прямой дороге водитель прекращает занос поворотом руля в сторону заноса, то на повороте этим способом автомобиль может попасть в придорожную канаву. Поэтому, не теряя присутствия духа, надо попытаться вывести автомобиль из бокового заноса на повороте сильной подачей горючей смеси, направляя колеса по оси движения, т. е. в нужном направлении. Можно сказать с уверенностью, что, если это произойдет во второй половине поворота, когда колеса начинают поворачиваться на прямую дорогу, своевременная подача горючей смеси даст возможность вывести автомобиль из заноса.

Приближаясь к повороту, водитель должен рассчитать, с какой скоростью можно пройти поворот. Чем круче поворот, хуже сцепление колес с дорогой (мокрая, скользкая дорога), больше вес автомобиля, выше расположен центр тяжести машины, тем меньше должна быть скорость автомобиля.

Правила безопасности движения обязывают водителя при приближении к закруглениям дороги заблаговременно снижать скорость и на поворотах двигаться на пониженной скорости.

При поворотах на большой угол надо помнить, что автомобиль при повороте занимает больше места, чем при движении в прямом направлении. Поэтому, особенно при поворотах в левую сторону, надо не затягивать поворот и не мешать проезду другого транспорта. Радиус поворота передних колес должен быть настолько большим, чтобы задние колеса, поворачивающиеся по дуге меньшего радиуса, не наехали на препятствие.

Поворот будет совершен технически правильно, если задние колеса при повороте в правую сторону пройдут на одинаковом расстоянии от закругления дороги, а при повороте в левую сторону — от центра поворота.

 

  1. Зеркала заднего вида.

Зеркала заднего вида – это зеркала, установленные на автомобиле и позволяющие водителю видеть происходящее сзади его транспортного средства (далее ТС) и по бокам от него. Существует два вида зеркал заднего вида: салонное зеркало и боковые зеркала. Салонное зеркало позволяет водителю наблюдать за происходящим за автомобилем, а боковые зеркала, в свою очередь, дают возможность оценить обстановку слева и справа от ТС.

    1. Отсутствие зеркал заднего вида.

В ПДД написано, что эксплуатация ТС без зеркал заднего вида запрещена. Это обусловлено рядом факторов.

При отсутствие салонного зеркала, у автолюбителя нет возможности оценить происходящее за автомобилем, то есть ТС, движущиеся за ним попадают в слепую зону относительно данного водителя. Из чего следует, что при совершении какого-либо маневра, водителю будет необходимо оглядеться по сторонам. Выполняя это действие, он неминуемо отвлечется от дороги и впереди идущего транспорта, что повышает риск дорожно-транспортного происшествия (далее ДТП).

Боковые зеркала дают возможность оценить ситуацию на дороге при совершении маневра (обгоне, перестроении, торможении и т.д.). Отсутствие боковых зеркал заднего вида значительно увеличивает мертвую зону (не просматриваемая зона около автомобиля). Это приводит к увеличению возможности возникновения аварийной ситуации. Стоит отметить, что в России, а также в странах с правосторонним движением, более необходимо, по сравнению с другим, левое боковое зеркало, так как по правую сторону от автомобиля как правило обочина, а обгоны и другие маневры выполняются с левой стороны от автомобиля.

    1. Виды зеркал.

Как известно из курса оптики, существует три типа зеркал: прямые, вогнутые и выпуклые.

  1. Прямые зеркала (см. рис. 6). По определению это плоская поверхность, зеркально отражающая свет. Изображение предмета, даваемое плоским зеркалом, формируется за счет лучей, отраженных от зеркальной поверхности. Это изображение является мнимым, так как оно образуется пересечением не самих отраженных лучей, а их продолжений в «зазеркалье».

Вследствие закона отражения света мнимое изображение предмета располагается симметрично относительно зеркальной поверхности. Размер изображения равен размеру самого предмета.

Изображение в плоском зеркале воображаемое, прямое (не обратное), одинаковое по размерам с предметом и расположено на таком же расстоянии от зеркала, что и сам предмет. 

Рис. 6.

  1. Вогнутые зеркала (см. рис. 7). Зеркало является вогнутым, если его центр находится дальше от смотрящего, чем его края, то есть отражающей поверхностью служит внутренняя сторона сферического сегмента.

Если говорить о изображении, которое дает вогнутое зеркало, то вне зависимости от положения отражаемого предмета, изображение будет действительное перевернутое. Что касается размера изображения, то он меняется в зависимости от положения отражаемого предмета относительно центра и фокуса. Исключением является случай, когда предмет находится на меньшем расстоянии, чем длина фокуса. В этом случае изображение отражаемого предмета будет мнимое, прямое и увеличенное.

Рис. 7

  1. Выпуклые зеркала. Зеркало является выпуклым, если его центр находится ближе к смотрящему, чем его края, то есть отражающей поверхностью служит наружная сторона сферического сегмента. Изображение в выпуклом зеркале — мнимое, прямое, уменьшенное, находится по другую сторону зеркала от предмета.

Рис. 8

    1. Выбор автомобильных зеркал.

При выборе автомобильных боковых зеркал заднего вида лучше остановиться на прямых или вогнутых зеркалах. Прямые зеркала позволяют реально увидеть сложившую ситуацию на дороге за автомобилем без уменьшения или увеличения изображения, но при этом будет незначительно ограничена видимость. Если рассматривать вогнутые зеркала, то в результате их использования размер мертвой зоны справа и слева уменьшается, а значит, видимость становится лучше и движение становится более безопасным.

Салонное же зеркало следует выбрать прямое или выпуклое. Выпуклое зеркало охватывает большее пространство и в силу этого позволяет водителю делать меньше лишних движений, таких как оборачиваться, оглядываться и так далее.

 

  1. Буксировка автомобиля.

4.1. Правила буксировки

— Перевозка на буксире требует  наличия у автомобиля-тягача  вспомогательных технических средств и оборудования, а также достаточно высокой квалификации водителя.

-На автомобиле-тягаче должен  быть включен ближний свет, а на буксируемом автомобиле — аварийная сигнализация (если она неисправна, то крепится знак «Аварийная остановка»).

— Запрещена перевозка пассажиров в кабине буксируемого и буксирующего автомобилей в связи со снижением их эксплуатационной надёжности.

— В качестве буксиров-тягачей  запрещено использовать мотоциклы.

— Скорость буксирования не должна превышать 50 км/ч (для автомобилей с механической трансмиссией) и 30 км/ч (для автомобилей с АКПП).

— Для автомобилей с АКПП также  имеются ограничения на протяженность пути буксировки до 30-80 км., если колеса буксируемого автомобиля «едут» по дороге. При этом рычаг переключения скоростей должен быть установлен в положение «N» (нейтраль).  Это ограничение вызвано конструктивными особенностями автоматической трансмиссии во избежание перегрева трансмиссионного масла.

Перевозка автомобиля на буксире может осуществляться несколькими способами. Рассмотрим способы буксировки подробнее.

    1. Буксировка на жесткой сцепке

В роли связующего элемента в этом случае выступает специальная металлическая тяга цилиндрической формы, либо буксировочная конструкция из двух тяг, соединенных под углом («треугольник»).

Данный вид буксировки не подходит для перемещения легковых автомобилей, потому что в их конструкции не предусмотрены места для крепления довольно увесистой жесткой тяги.

Правила буксировки на жесткой сцепке:

— Длина буксировочной тяги не должна превышать 4 метров.

— Водитель буксируемого средства должен находиться за рулем, чтобы придерживаться траектории движения, однако при сцепке типа «треугольник» это необязательно (хотя желательно).

— На жесткой сцепке запрещена буксировка автомобилей с неисправным рулевым управлением.

— Буксировка на жесткой сцепке допустима в гололедицу, при движении на скользкой дороге.

 

    1. Буксировка на гибкой сцепке

Для того, чтобы осуществить буксировку на гибкой сцепке, следует использовать капроновый или стальной трос в качестве связующего звена. Кстати, капроновый фал ни в чем не уступает стальному (например, по прочности). Под нагрузкой капроновый фал начинает растягиваться. Благодаря этому, вероятность повредить детали кузова уменьшается, а обрыв или деформация буксирных проушин становятся практически невозможными.

Но и у капронового троса есть недостаток: в том случае, если он сильно провиснет, то, скорее всего, он сотрется об асфальт. Правда, сейчас этот «минус» ликвидирован, благодаря созданию специальной защитной оболочки для капронового троса. Сама оболочка тоже является достаточно гибкой. Применение такой конструкции не допускает провисание капронового фала (даже в тех случаях, когда между автомобилями образовывается минимальное расстояние в 1,5-2 метра).

Впереди крутой поворот.

Авто Советы продолжают актуальную тему безаварийного вождения.  В каждой из статей мы рассказываем об одном из важных штрихов в технике безопасного вождения автомобиля. Постепенно из этих штрихов сложится целостная картина, которую можно будет назвать ” Как не попасть в ДТП!”   В качестве наглядных примеров используются видеоролики реальных дорожно-транспортных происшествий.  Их сегодня выложено в интернете огромное количество. Мы тщательно подбираем именно те ситуации, которые помогают правильно понять причины происшествия, чтобы не повторять этих ошибок в дальнейшем. Сегодня речь пойдёт о прохождении поворотов на загородных трассах.

Поворот дороги – это очень ответственный участок для водителя автомобиля. Особенно зимой.  Согласно закону механики на повороте на автомобиль действует центробежная сила, которая стремиться выбросить авто с дороги. Посмотрите на рисунок. Здесь изображён схематический поперечный разрез гоночного автомобиля на повороте. Предполагается, что это вид сзади и автомобиль совершает правый поворот. При этом полотно дороги наклонено наружу по отношении к повороту, хотя по строительным нормам наклон должен быть в противоположную сторону.

То есть, здесь изображён самый неблагоприятный случай, который на дорогах бывшего Союза встречается довольно часто. Центр тяжести автомобиля обозначен О, масса автомобиля –  G , С – центробежная сила и R – результирующая сила действующая на автомобиль. На рисунке показан тот случай, когда результирующая сила приходится на самую левую точку опоры.

Ещё небольшое смещение этой силы влево  и она выходит за пределы колеи, опоры автомобиля и он перевернётся. Это превышение может быть как из-за большего наклона полотна дороги, так и из-за увеличения центробежной силы С. Если же полотно дороги горизонтальное и  результирующая сила R не будет выходить за пределы колеи, то увеличение силы С будет приводить к скольжению автомобиля по дороге, автомобиль будет выносить с дорожного полотна. Используя общепринятые обозначения, центробежная сила Fц  равна  Fц = mV2/R .  То есть, она прямо пропорциональна массе авто m, пропорциональна квадрату скорости v и обратно пропорциональна радиусу поворота R.

Для тех, кому не понятна физика скажу попроще.  Самое сильное влияние на центробежную силу оказывает скорость. Увеличение скорости в два раза, увеличивает центробежную силу в четыре раза! Поэтому перед поворотом скорость должна быть снижена до такого предела, чтобы автомобиль не опрокинулся и его не вынесло с дороги. А теперь перейдём к конкретным примерам. Как Вы увидите перевернутся можно на сухой дороге и средь белого дня.

Кстати говоря бензовоз ехал со скоростью всего 65 км/час. Почему  же он перевернулся, точнее, лёг на бок?  Как следует из рисунка на устойчивость на повороте кроме всего прочего влияет расположение центра тяжести. Чем он выше, тем менее устойчив автомобиль. Например, гружёный багажник на крыше автомобиля должен учитываться водителем при прохождении поворота.  Ясно, что у гружёного бензовоза центр тяжести поднят. Но здесь действовал ещё один коварный фактор незаметный внешне. Цистерна была заполнена не полностью!

И на повороте груз (бензин) смещался во внешнюю сторону поворота, смещая, естественно и центр тяжести.  Отсюда сразу напрашивается один из выводов. Груз надо размещать как можно ниже к дороге и обязательно крепить. Если это невозможно (как в данном случае), то надо снижать скорость перед поворотом.

Вот ещё характерный случай.

Здесь как раз полотно дороги явно наклонено во внешнюю сторону дороги и, скорее всего груз в автомобиле расположен не по центру кузова, а справа. Законы механики не обманешь!

Третий характерный эпизод. Грузовик свалился на легковой автомобиль.!

Избегайте езду в параллельных рядах на повороте с грузовиком!

При выходе из поворота нужно плавно увеличиваем скорость – это способствует большей устойчивости автомобиля.

Четвёртый эпизод, когда на повороте автомобиль занесло, так как водитель не снизил скорость.

Если бы скорость была снижена, то автомобиль бы не занесло.

А теперь посмотрите на зимний сюжет, обратите внимание на скорость автомобиля, её пишет видеорегистратор. Достаточно посмотреть 31 секунду, далее ничего интересного.

Хороших Вам зимних дорог и удачи!

Об авторе

Как все движется и почему: дураки

Я впервые узнал в школе физики о мифе о центробежной силе. На бумаге и в теории это очень реальная сила, которая отталкивает вас от чего-то. Этот термин используется в автомобильном мире, когда люди обсуждают силы, действующие на автомобиль, когда он едет за угол. Дело в том, что это неправда. Даже не близко. Не будь дураком, который думает иначе. Действующие силы — это центростремительные силы. Силы втягивают вас внутрь, а не наоборот.Позволь мне объяснить.

Я начну с очень технического определения из моего восьмого издания Engineer-In-Training Reference Manual Майкла Р. Линдебурга, PE. В книге говорится: «Второй закон Ньютона гласит, что существует сила для каждого ускорения, которое испытывает тело. Для тела, движущегося по криволинейной траектории, общее ускорение можно разделить на тангенциальную и нормальную составляющие. По второму закону Ньютона существуют соответствующие силы в касательном и нормальном направлениях.Сила, связанная с нормальным направлением, известна как центростремительная сила ».

Линдебург продолжает: «Центростремительная сила — это действительная сила , действующая на тело по направлению к центру вращения. Так называемая центробежная сила — это кажущаяся сила на тело , направленная от центра вращения ». Я добавил курсив для акцента. Давайте разберемся с этим.

Нормальная сила, которую упоминает Линдебург, относится к силе, втягивающей тело внутрь, центростремительной силе.А тело в данном случае — это машина. Когда автомобиль поворачивает, сила трения между шиной и дорогой создает силу, тянущую вас за угол или к центру. Это сила трения, которая позволяет машине поворачиваться при движении.

Сложность всего этого проистекает из третьего закона физики Ньютона: на каждое действие должно быть равное и противоположное противодействие. Поскольку трение между шинами и дорогой втягивает машину, вы, человек в машине, чувствуете сопротивление этому движению, поэтому вы чувствуете тягу наружу.Вот почему, когда вы поворачиваете налево, ваше тело хочет пойти направо и наоборот. Ваше тело реагирует на приложенную силу, которая втягивает вас внутрь.

Этот контент импортирован с YouTube. Вы можете найти тот же контент в другом формате или найти дополнительную информацию на их веб-сайте.

Лучший способ подумать об этом интуитивно — это то, что происходит с автомобилем, когда сила трения между дорогой и шиной исчезает. Возможно, вы ударились о кусок льда, или шина просто потеряла сцепление с дорогой.Каждый раз, когда это происходит, машина перестает завернуть за угол. Он просто продолжает двигаться прямо с того места, где потерял хватку, пока не остановится. Сила, втягивающая машину, перестала действовать на машину.

Если бы центробежная сила действительно действовала на автомобиль, что бы произошло, если бы она ушла? Вы бы сильнее загнали в угол! Автомобиль продолжал въезжать, потому что не было «центробежной силы», которая могла бы его остановить. Этого не происходит, потому что нет центробежной силы. Всезнайка-инженер рассуждала.

Посмотрите видео выше, чтобы понять, что я имею в виду.

Справочное руководство для начинающих инженеров: 8-е (восьмое) издание

Этот контент создается и поддерживается третьей стороной и импортируется на эту страницу, чтобы помочь пользователям указать свои адреса электронной почты. Вы можете найти больше информации об этом и подобном контенте на сайте piano.io.

Требование центростремительной силы

Как упоминалось ранее в этом уроке, объект, движущийся по кругу, испытывает ускорение.Даже если двигаться по периметру круга с постоянной скоростью, все равно происходит изменение скорости и, как следствие, ускорение. Это ускорение направлено к центру круга. И в соответствии со вторым законом движения Ньютона, объект, испытывающий ускорение, должен также испытывать чистую силу. Направление чистой силы совпадает с направлением ускорения. Итак, для объекта, движущегося по кругу, на него должна действовать внутренняя сила, вызывающая его внутреннее ускорение.Это иногда называют требованием центростремительной силы . Слово центробежный (не путать с F-словом центробежный ) означает поиск центра. Для кругового движения объекта существует результирующая сила, действующая к центру, которая заставляет объект искать центр.

Чтобы понять важность центростремительной силы, важно иметь четкое представление о первом законе движения Ньютона — закон инерции .Закон инерции гласит, что …

… движущиеся объекты имеют тенденцию оставаться в движении с той же скоростью и в том же направлении, если на них не действует неуравновешенная сила.

Согласно первому закону движения Ньютона, это естественная тенденция всех движущихся объектов продолжать движение в том же направлении, в котором они движутся … если на объект не действует какая-то неуравновешенная сила, отклоняющая его движение от его прямого направления. -линейный путь.Движущиеся объекты будут естественно двигаться по прямым линиям; неуравновешенная сила требуется только для того, чтобы заставить его повернуться. Таким образом, для движения объектов по кругу требуется наличие неуравновешенной силы.


Инерция, сила и ускорение для пассажира автомобиля

Идея, выраженная законом инерции Ньютона, не должна нас удивлять. Мы сталкиваемся с этим феноменом инерции почти каждый день, когда водим автомобиль.Например, представьте, что вы пассажир в машине на светофоре. Индикатор загорится зеленым, и водитель начнет ускоряться, находясь в состоянии покоя. Автомобиль начинает ускоряться вперед, но относительно сиденья, на котором вы находитесь, начинает наклоняться назад. Ваше тело в состоянии покоя имеет тенденцию оставаться в покое. Это один из аспектов закона инерции — «покоящиеся объекты стремятся оставаться в покое». Когда колеса автомобиля вращаются, создавая прямую силу на машине и вызывая ускорение вперед, ваше тело стремится оставаться на месте.Вам определенно может показаться, что ваше тело испытывает обратную силу, заставляющую его ускоряться в обратном направлении. Тем не менее, вам будет сложно определить такую ​​обратную силу на вашем теле. На самом деле нет ни одного. Ощущение отбрасывания назад — это просто тенденция вашего тела сопротивляться ускорению и оставаться в состоянии покоя. Автомобиль ускоряется из-под вашего тела, оставляя у вас ложное ощущение, что вас толкают назад.

Теперь представьте, что вы находитесь в той же машине, движущейся с постоянной скоростью, приближаясь к светофору.Водитель нажимает на тормоза, колеса машины блокируются, и машина начинает заносить до полной остановки. На движущийся вперед автомобиль действует сила, направленная назад, а затем на автомобиль происходит ускорение назад. Однако ваше тело, находясь в движении, имеет тенденцию продолжать движение, пока машина буксует до полной остановки. Вам наверняка может показаться, что ваше тело испытывает силу, направленную вперед, заставляя его ускоряться вперед. Тем не менее, вам снова будет трудно определить такую ​​прямую силу на вашем теле.На самом деле нет физического объекта, ускоряющего вас вперед. Ощущение того, что вас выбрасывает вперед, — это просто тенденция вашего тела сопротивляться замедлению и оставаться в состоянии поступательного движения. Это второй аспект закона инерции Ньютона — «движущийся объект стремится оставаться в движении с той же скоростью и в том же направлении …». Неуравновешенная сила, действующая на автомобиль, заставляет автомобиль замедляться, в то время как ваше тело продолжает движение вперед. Вы снова остаетесь с ложным ощущением, что вас толкают в направлении, противоположном вашему ускорению.

Эти два сценария вождения представлены на следующем рисунке.

В каждом случае — трогание автомобиля с места и торможение движущегося автомобиля до остановки — направление, на которое наклоняются пассажиры, противоположно направлению ускорения. Это просто результат инерции пассажира — тенденции сопротивляться ускорению. Наклон пассажира — это не ускорение само по себе, а, скорее, тенденция поддерживать состояние движения, пока автомобиль ускоряется.Тенденция тела пассажира поддерживать состояние покоя или движения, в то время как окружающая среда (автомобиль) ускоряется, часто неверно истолковывается как ускорение. Это становится особенно проблематичным, когда мы рассматриваем третий возможный инерционный опыт пассажира в движущемся автомобиле — левый поворот.

Предположим, что на следующем этапе вашего пути водитель автомобиля делает резкий поворот налево с постоянной скоростью. Во время поворота машина движется по круговой траектории.То есть машина заметает четверть круга. Сила трения, действующая на повернутые колеса автомобиля, вызывает несбалансированную силу на автомобиль и последующее ускорение. Неуравновешенная сила и ускорение направлены к центру круга, вокруг которого поворачивается автомобиль. Однако ваше тело находится в движении и имеет тенденцию оставаться в движении. Именно инерция вашего тела — тенденция сопротивляться ускорению — заставляет его продолжать движение вперед. Пока машина ускоряется внутрь, вы продолжаете движение по прямой.Если вы сидите с пассажирской стороны автомобиля, то в конечном итоге внешняя дверь автомобиля ударит вас, когда машина повернет внутрь. Это явление может заставить вас думать, что вы ускоряетесь от центра круга. На самом деле вы продолжаете свой прямой инерционный путь, касающийся окружности, в то время как машина ускоряется из-под вас. Ощущение внешней силы и внешнего ускорения — ложное ощущение. Нет физического объекта, способного вытолкнуть вас наружу.Вы просто испытываете тенденцию вашего тела продолжать свой путь, касающийся круговой траектории, по которой поворачивает автомобиль. Вы снова остаетесь с ложным ощущением, что вас толкают в направлении, противоположном вашему ускорению.

Центростремительная сила и изменение направления

Любой объект, движущийся по кругу (или по круговой траектории), испытывает центростремительную силу .То есть существует некоторая физическая сила, толкающая или притягивающая объект к центру круга. Это требование центростремительной силы. Слово центростремительный — это просто прилагательное, используемое для описания направления силы. Мы не вводим новый тип силы , а скорее описываем направление результирующей силы, действующей на объект, который движется по кругу. Каким бы ни был объект, если он движется по кругу, на него действует некоторая сила, которая заставляет его отклоняться от своего прямолинейного пути, ускоряться внутрь и двигаться по круговой траектории.Ниже показаны три таких примера центростремительной силы.

Когда автомобиль совершает поворот, сила трения, действующая на повернутые колеса автомобиля, создает центростремительную силу, необходимую для кругового движения.

Когда ведро с водой привязано к веревке и вращается по кругу, сила натяжения, действующая на ведро, обеспечивает центростремительную силу, необходимую для кругового движения.

Когда Луна вращается вокруг Земли, сила тяжести, действующая на Луну, обеспечивает центростремительную силу, необходимую для кругового движения.

Центростремительная сила для равномерного кругового движения изменяет направление объекта без изменения его скорости. Идея о том, что неуравновешенная сила может изменить направление вектора скорости, но не его величину, может показаться немного странной.Как такое могло быть? Есть несколько способов подойти к этому вопросу. Один из подходов включает анализ движения с точки зрения работы-энергии. Вспомните из блока 5 Физического класса, что работает сила , действующая на объект, вызывая смещение . Объем работы, проделанной над объектом, находится с помощью уравнения

Работа = Сила * смещение * косинус (Тета)

, где Theta в уравнении представляет собой угол между силой и смещением.Поскольку центростремительная сила действует на объект, движущийся по кругу с постоянной скоростью, сила всегда действует внутрь, поскольку скорость объекта направлена ​​по касательной к окружности. Это означало бы, что сила всегда направлена ​​перпендикулярно направлению смещения объекта. Угол Theta в приведенном выше уравнении равен 90 градусам, а косинус 90 градусов равен 0. Таким образом, работа, совершаемая центростремительной силой в случае равномерного кругового движения, равна 0 Джоулей. Вспомните также из Раздела 5 Класса физики, что, когда над объектом не работают внешние силы, общая механическая энергия (потенциальная энергия плюс кинетическая энергия) объекта остается постоянной.Таким образом, если объект движется по горизонтальному кругу с постоянной скоростью, центростремительная сила не работает и не может изменить общую механическую энергию объекта. По этой причине кинетическая энергия и, следовательно, скорость объекта останутся постоянными. Сила действительно может ускорить объект, изменив его направление, но не может изменить его скорость. Фактически, всякий раз, когда неуравновешенная центростремительная сила действует перпендикулярно направлению движения, скорость объекта остается постоянной.Чтобы неуравновешенная сила изменила скорость объекта, должна быть составляющая силы в направлении (или противоположном) направлении движения объекта.

Применение векторных компонентов и второго закона Ньютона

Второй подход к этому вопросу о том, почему центростремительная сила вызывает изменение направления, но не изменение скорости, включает компоненты вектора и второй закон Ньютона.Следующий воображаемый сценарий будет использован, чтобы проиллюстрировать эту мысль.

Предположим, что на местной ледяной фабрике кусок льда выскользнул из морозильной камеры, и механический рычаг приложил силу, чтобы ускорить его по ледяной поверхности, свободной от трения. На прошлой неделе механическая рука вышла из строя и произвольно давила на себя. Ниже показаны различные направления сил, действующих на движущуюся глыбу льда. В каждом случае наблюдайте за силой по сравнению с направлением движения ледяного блока и прогнозируйте, будет ли сила ускоряться, замедляться или не влиять на скорость блока.Используйте векторные компоненты, чтобы делать свои прогнозы. Затем проверьте свои ответы, нажав на кнопку.

Физическое положение Разгоняться, замедляться или не влиять на скорость? Пояснение

а.

г.

г.

г.

e.

Приведенные выше примеры показывают, что сила способна замедлить или ускорить объект, только когда есть компонент, направленный в том же или противоположном направлении, что и движение объекта. В случае е вертикальная сила не изменяет горизонтальное движение.Иногда говорят, что перпендикулярные компоненты движения не зависят друг от друга. Вертикальная сила не может повлиять на горизонтальное движение.


Подводя итог, объект при равномерном круговом движении испытывает внутреннюю чистую силу. Эту внутреннюю силу иногда называют центростремительной силой, где центростремительная сила описывает ее направление. Без этой центростремительной силы объект никогда не мог бы изменить свое направление. Тот факт, что центростремительная сила направлена ​​перпендикулярно тангенциальной скорости, означает, что сила может изменять направление вектора скорости объекта без изменения его величины.

Мы хотели бы предложить … Иногда просто прочитать об этом недостаточно. Вы должны взаимодействовать с ним! И это именно то, что вы делаете, когда используете один из интерактивных материалов The Physics Classroom. Мы хотели бы предложить вам совместить чтение этой страницы с использованием наших интерактивных элементов Uniform Circular Motion Interactive и / или Race Track Interactive. Вы можете найти их в разделе Physics Interactives на нашем веб-сайте.Оба интерактивных модуля позволяют учащемуся интерактивно исследовать чистую силу для объекта, движущегося по кругу.


Проверьте свое понимание

Для вопросов №1– №5: Объект движется по по часовой стрелке по кругу с постоянной скоростью. Используйте свое понимание концепций скорости, ускорения и силы, чтобы ответить на следующие пять вопросов.Используйте диаграмму, показанную справа. Нажмите кнопку, чтобы проверить свои ответы.

1. Какой вектор ниже представляет направление вектора силы, когда объект находится в точке A на окружности?

2. Какой вектор ниже представляет направление вектора силы, когда объект находится в точке C на окружности?

3. Какой вектор ниже представляет направление вектора скорости, когда объект находится в точке B на окружности?

4.Какой вектор ниже представляет направление вектора скорости, когда объект находится в точке C на окружности?

5. Какой вектор ниже представляет направление вектора ускорения, когда объект находится в точке B на окружности?


6. Рекс Вещи и Дорис заперта на свидание. Рекс быстро поворачивает направо.Дорис начинает скользить по виниловому сиденью (которое Рекс предварительно отполировал и отполировал) и сталкивается с Рексом. Чтобы преодолеть неловкость ситуации, Рекс и Дорис начинают обсуждать физику только что испытанного движения. Рекс предполагает, что объекты, движущиеся по кругу, испытывают внешнюю силу. Таким образом, когда поворот был сделан, Дорис испытала внешнюю силу, которая подтолкнула ее к Рексу. Дорис не соглашается, утверждая, что объекты, движущиеся по кругу, испытывают внутреннюю силу. В этом случае, по словам Дорис, Рекс двигался по кругу из-за того, что дверь толкала его внутрь.Дорис не двигалась по кругу, поскольку не было силы, толкающей ее внутрь; она просто продолжала двигаться по прямой, пока не столкнулась с Рексом. Кто прав? Аргументируйте одну из этих двух позиций.


7. Кара Лотт тренируется в зимнем вождении на стоянке GBS. Кара поворачивает руль, чтобы повернуть налево, но ее машина продолжает движение по льду по прямой. Учитель A и учитель B наблюдали за этим явлением.Учитель А утверждает, что отсутствие силы трения между шинами и льдом приводит к балансу сил, который заставляет автомобиль двигаться по прямой. Учитель Б утверждает, что лед оказывал внешнюю силу на шину, чтобы уравновесить поворотную силу и, таким образом, удерживать машину, движущуюся по прямой. Какой учитель (А или Б) учитель физики? ______ Объясните ошибочность аргумента другого учителя.


Что такое центростремительные силы и центробежная аренда?

Центростремительный означает «поиск центра» или «к центру» — когда объект движется по круговой траектории вокруг фиксированного центра, создается сила, называемая центростремительной силой, которая толкает объект к центру.

Без центростремительной силы объект не смог бы перемещаться по кругу вокруг фиксированного центра.

Пример: сила, которая заставляет планету продолжать вращаться вокруг Солнца.

Центробежная сила (иногда называемая центробежной силой) работает противоположно центробежной силе.
Центробежность — это тенденция объекта, который движется по круговой траектории, к желанию двигаться в прямом направлении.

На самом деле «центробежная сила» — это тенденция, создаваемая инерцией для любого движущегося объекта, стремящегося двигаться в том же направлении и с той же скоростью, поэтому при движении по кругу объект хочет уйти из круга и двигаться в прямом направлении. .

Если вы привяжете объект к веревке, а затем повернете его по кругу вокруг себя, удерживать объект станет труднее. Центробежная тенденция толкает объект в прямом направлении или наружу, что затрудняет удержание.Чем быстрее вы вращаете объект, тем выше центробежное владение.

Итак, при движении по круговой или криволинейной траектории центростремительная сила — это сила, которая дает вам возможность продолжать движение по кругу, а центробежная тенденция — это сила инерции, которая уводит ваше транспортное средство с круговой траектории в сторону прямая линия.

Когда вы едете по круговой или криволинейной траектории, соблюдайте следующие правила, чтобы противодействовать центробежной тенденции:

  1. Снизьте скорость перед тем, как начать движение по повороту.
  2. Не тормозите резко или резко при повороте, иначе вы потеряете контроль над автомобилем. Безопасно снижайте скорость, постепенно нажимая на тормоза при повороте.
  3. Безопасно переключитесь на более низкую передачу, чтобы сбавить скорость.

Некоторые современные дороги имеют наклон (проезжая часть с наклоном), что обеспечивает лучший контроль поворота во время поворотов; это функция, которая противодействует центробежной тенденции, которая уводит автомобили от поворотов. Однако независимо от того, едете ли вы по крутому или плоскому повороту, всегда снижайте скорость, когда начинаете движение по круговой или изогнутой дорожке.

Физика и естественные законы вождения | Привычки и советы по безопасному вождению

19. Формулы успеха

Как быстро я еду?

Возьми свою скорость, добавь 1/2 своей скорости. Результат — ваша скорость в футах в секунду.

S + 1 / 2S = S (футов в секунду)

Сколько времени потребуется, чтобы остановиться?

Умножьте свою скорость на 1/10 скорости и разделите результат на два.Результат — ваш тормозной путь после нажатия на педаль тормоза футов.

(S x 1 / 10S) / 2 = SD (футы)

Сколько времени это действительно займет?

Поскольку на распознавание опасности и последующее реагирование на нее требуется около одной секунды, добавьте расстояние, которое вы пройдете за одну секунду, к тормозному пути, чтобы найти общее расстояние в футах, которое потребуется вам до остановки.

(S + 1 / 2S) + (S x 1/10 S / 2) = общий тормозной путь (футы)

Что такое кинетическая энергия и как она влияет на меня?

Кинетическая энергия или импульса влияет на все аспекты вождения.Чтобы остановить движущееся транспортное средство, вы должны рассеять (избавиться от) кинетическую энергию за счет трения тормозов или удара о другой объект (так называемая сила удара). Чтобы управлять автомобилем, вы должны преодолеть кинетическую энергию транспортного средства (оно хочет продолжать движение по прямой), чтобы изменить направление.

KE = 1 / 2M x V 2

Любое увеличение массы (веса) транспортного средства пропорционально увеличит KE .Удвоение массы удвоит массу KE .

Любое увеличение скорости (скорости) транспортного средства увеличит KE на квадрат изменения скорости. Удвоение , скорость будет вчетверо KE .

Благодаря торможению и уменьшению вашей скорости вдвое, KE теперь составляет всего 1/4 (1/4) от того, что было на вашей исходной скорости. Чем меньше KE, тем меньше повреждений будет нанесено вам и вашему автомобилю в результате аварии.

Что такое коэффициент трения?

Это количество тяги , обеспечиваемое «следом» четырех шин, контактирующих с дорогой. Четыре совершенно новых шины на идеально гладкой поверхности дают коэффициент CF 1.0; на гладком, сухом асфальте КФ составляет 0,9; на мокром асфальте или талом льду CF составляет всего 0,35; мерзлый асфальт дает CF 0,20; а лед имеет CF всего 0,05.

Центростремительная сила, центробежная сила — в чем дело?

Несколько комментаторов выразили озабоченность по поводу использования центробежной силы после моей напыщенной речи об использовании слова «сила».Итак, в чем же дело с этими двумя условиями? Их можно использовать? Это настоящие силы?

Во-первых, это реальные силы?

Это зависит от того, что вы подразумеваете под реальным. Что такое сила? Вот краткий обзор того, что такое сила. Ранее я говорил о реальных и нереальных силах. Что касается меня, я говорю, что если сила, по сути, является одной из 4 фундаментальных сил, то она «реальна». С этим определением центростремительная сила будет реальной, а центробежная — нереальной.

Центростремительная сила

Центростремительная сила — это сила, необходимая для того, чтобы что-то двигалось по кругу.Что ж, может быть, было бы лучше сказать, что центростремительная сила — это сила, необходимая для того, чтобы что-то изменить направление. Центростремительная сила постоянной величины, которая всегда перпендикулярна направлению движения, заставит объект двигаться по кругу.

Важно отметить, что центростремительная сила — это еще некоторая сила. Например, предположим, что вы качаете камень вокруг головы с помощью веревки. На камень действует сила, заставляющая его двигаться по кругу. Эта сила представляет собой натяжение струны (которое является реальной фундаментальной силой, основанной на электростатических взаимодействиях между атомами).В этом случае натяжение будет центростремительной силой. Другой пример — машина движется по круговой траектории. В этом случае существует центростремительная сила, это сила трения между шинами и дорогой. В обоих этих примерах, если вы уберете центростремительную силу (сломаете веревку или двинетесь по льду), объект будет двигаться по прямой линии.

В большинстве учебников термин центростремительная сила используется правильно. Однако на самом деле это не нужно. Вместо этого они могут также относиться к центростремительному ускорению как к ускорению объекта, движущегося по кругу.Большинство учебников пишут это так:

Эта форма не включает векторную природу ускорения. Направление этого ускорения всегда должно быть направлено к центру круга.

Центробежная сила

Этот термин гораздо реже встречается во вводных текстах по физике. Это не реальная сила (согласно моему определению реальной силы выше). Однако он играет полезную роль в некоторой физике. Позвольте мне начать с примера. Допустим, мальчик (или девочка, это действительно могла быть и девочка) сидел на неподвижной карусели с мячом.Затем он перекатывает мяч девушке на другой стороне. Нет проблем, правда? Кстати о каруселях, они просто больше не кладут эти вещи в парки, не так ли? Во всяком случае, вот диаграмма.

В этом случае, когда карусель не вращается, мяч летит прямо к девушке. Без проблем. Теперь предположим, что карусель крутится. Если мальчик попытается перекатить мяч прямо к девушке, мяч действительно все равно будет идти по прямой (с учетом небольших сил трения), если смотреть со стороны человека, не участвующего в карусели.Мяч не доберется до девушки, потому что она развернется в сторону.

Мальчик и девочка, сидящие на вращающейся карусели, увидят, что мяч движется не по прямой (в их системе отсчета карусели). Вот две диаграммы, которые могут помочь.

Проблема с мальчиком и девочкой на карусели. Они видят, что мяч НЕ летит по прямой. Поскольку они понимают силу и движение, они говорят: «Эй, если он не движется по прямой линии, на него должна быть сила».Они правы. Однако они не находятся в инерциальной системе отсчета. Их рамка на самом деле ускоряется (потому что движется по кругу). Чтобы все получилось в неинтертициальном кадре, необходимо добавить силу (нереальную силу). Эта сила и есть центробежная сила.

Центробежная сила — это сила (нереальная сила), которая необходима для того, чтобы все работало так, как вы могли бы подумать в ускоряющейся системе отсчета. Лучший пример — это когда вы находитесь в машине, которая поворачивает.Если машина поворачивает налево, ОЩУЩЕНИЕ дает вам силу, толкающую вас вправо (в вашей системе координат внутри машины). Это правильно было бы назвать центробежной силой. Единственная реальная проблема заключается в том, что если вы назовете центробежную силу реальной (как при фундаментальном взаимодействии) силой.

Итак, в конце концов, большинству людей не нужен ни один из этих терминов. Из этих двух центробежный является более необходимым, но также с большей вероятностью будет использоваться неправильно.

6.3 Центростремительная сила — Университетская физика, том 1

Цели обучения

К концу раздела вы сможете:

  • Объясните уравнение центростремительного ускорения
  • Примените второй закон Ньютона, чтобы получить уравнение для центростремительной силы
  • Используйте концепции кругового движения при решении задач, связанных с законами движения Ньютона

В «Движении в двух и трех измерениях» мы изучили основные концепции кругового движения.Объект, совершающий круговое движение, например один из гоночных автомобилей, показанных в начале этой главы, должен ускоряться, потому что он меняет направление своей скорости. Мы доказали, что это центрально направленное ускорение, называемое центростремительным ускорением , выражается формулой

.

, где v — скорость объекта, направленная по касательной к кривой в любой момент времени. Если мы знаем угловую скорость

, тогда мы можем использовать

Угловая скорость показывает скорость, с которой объект поворачивает кривую, в рад / с.Это ускорение действует по радиусу криволинейной траектории и поэтому также называется радиальным ускорением.

Ускорение должно производиться силой. Любая сила или комбинация сил могут вызвать центростремительное или радиальное ускорение. Вот лишь несколько примеров: натяжение троса на тросовом шаре, сила притяжения Земли на Луне, трение между роликовыми коньками и полом катка, сила наклона проезжей части, действующая на автомобиль, и силы, действующие на трубу вращающейся центрифуги. . Любая чистая сила, вызывающая равномерное круговое движение, называется центростремительной силой .Направление центростремительной силы — к центру кривизны, то же самое, что и направление центростремительного ускорения. Согласно второму закону движения Ньютона, чистая сила равна массе, умноженной на ускорение:

Для равномерного кругового движения ускорением является центростремительное ускорение: .

Таким образом, величина центростремительной силы

это

Подставляя выражения для центростремительного ускорения

получаем два выражения для центростремительной силы

по массе, скорости, угловой скорости и радиусу кривизны:

Вы можете использовать любое более удобное выражение для центростремительной силы.Центростремительная сила

всегда перпендикулярно траектории и указывает на центр кривизны, потому что

перпендикулярно скорости и указывает на центр кривизны. Обратите внимание, что если вы решите первое выражение для r , вы получите

Это означает, что для данной массы и скорости большая центростремительная сила вызывает малый радиус кривизны, то есть резкую кривую, как на (Рисунок).

Рисунок 6.20. Сила трения обеспечивает центростремительную силу и численно равна ей. Центростремительная сила перпендикулярна скорости и вызывает равномерное круговое движение. Чем больше

, тем меньше радиус кривизны r и круче кривизна. Вторая кривая имеет то же v, но большее значение

.

дает меньшее r ‘.

Пример

Какой коэффициент трения нужен автомобилям на плоской кривой?

(a) Рассчитайте центростремительную силу, действующую на 900.Автомобиль массой 0 кг, который преодолевает поворот радиусом 500,0 м со скоростью 25,00 м / с. (b) Допуская кривую без наклона, найдите минимальный статический коэффициент трения между шинами и дорогой, при этом статическое трение является причиной, препятствующей скольжению автомобиля ((Рисунок)).

Рисунок 6.21 Этот автомобиль на ровной поверхности движется в сторону и поворачивает налево. Центростремительная сила, заставляющая автомобиль вращаться по круговой траектории, возникает из-за трения между шинами и дорогой. Требуется минимальный коэффициент трения, иначе автомобиль будет двигаться по кривой с большим радиусом и съезжать с проезжей части.
Стратегия
  1. Мы знаем, что

    Таким образом,

  2. (Рисунок) показывает силы, действующие на автомобиль на кривой без наклона (ровной поверхности). Трение направлено влево, предотвращая скольжение автомобиля, и поскольку это единственная горизонтальная сила, действующая на автомобиль, трение в данном случае является центростремительной силой. Мы знаем, что максимальное статическое трение (при котором шины катятся, но не скользят) составляет

    где

    — статический коэффициент трения, а Н, — нормальная сила.Нормальная сила равна массе автомобиля на ровной поверхности, поэтому

    Таким образом, центростремительная сила в данной ситуации равна

    .

    Теперь у нас есть связь между центростремительной силой и коэффициентом трения. Используя уравнение

    получаем

    Решаем это за

    , отмечая, что масса отменяется, и получаем

    Подстановка известных,

    (Поскольку коэффициенты трения являются приблизительными, ответ дается только двумя цифрами.)

Значение

Коэффициент трения, указанный на (Рисунок) (b), намного меньше, чем обычно наблюдается между шинами и дорогой. Автомобиль все еще движется по кривой, если коэффициент больше 0,13, потому что трение покоя является реактивной силой, которая может принимать значение меньше, но не больше

.

Более высокий коэффициент также позволит автомобилю преодолевать поворот на более высокой скорости, но если коэффициент трения меньше, безопасная скорость будет менее 25 м / с.Обратите внимание, что масса отменяется, подразумевая, что в этом примере не имеет значения, насколько сильно загружена машина для прохождения поворота. Масса сокращается, потому что трение считается пропорциональным нормальной силе, которая, в свою очередь, пропорциональна массе. Если бы поверхность дороги была наклонной, нормальная сила была бы меньше, как обсуждается далее.

Проверьте свое понимание

Автомобиль, движущийся со скоростью 96,8 км / ч, движется по круговой кривой радиусом 182,9 м по ровной проселочной дороге. Какой должен быть минимальный коэффициент статического трения, чтобы автомобиль не скользил?

[показать-ответ q = ”694795 ″] Показать решение [/ раскрыть-ответ]
[скрытый-ответ a =” 694795 ″] 0.40 [/ hidden-answer]

Кривые с наклоном

Давайте теперь рассмотрим кривых с наклоном , где уклон дороги помогает вам преодолевать кривую ((Рисунок)). Чем больше угол

, тем быстрее вы сможете пройти кривую. Например, гоночные трассы для велосипедов и автомобилей часто имеют крутые повороты. На «кривой с идеальным наклоном» угол

такова, что вы можете преодолевать поворот на определенной скорости без трения между шинами и дорогой.Получим выражение для

для кривой с идеальным наклоном и рассмотрим связанный с ней пример.

Рис. 6.22 Автомобиль на этой кривой с наклоном удаляется и поворачивает налево.

Для perfect bank чистая внешняя сила равна горизонтальной центростремительной силе в отсутствие трения. Составляющие нормальной силы Н в горизонтальном и вертикальном направлениях должны равняться центростремительной силе и массе автомобиля соответственно.В случаях, когда силы не параллельны, удобнее всего рассматривать компоненты вдоль перпендикулярных осей — в данном случае вертикального и горизонтального направлений.

(рисунок) показывает диаграмму свободного тела для автомобиля на кривой без трения с наклоном. Если угол

идеально подходит для скорости и радиуса, тогда чистая внешняя сила равна необходимой центростремительной силе. Единственными двумя внешними силами, действующими на автомобиль, являются его вес

.

и нормальная сила дороги

(Поверхность без трения может оказывать только силу, перпендикулярную поверхности, то есть нормальную силу.Эти две силы должны складываться, чтобы получить чистую внешнюю силу, горизонтальную по направлению к центру кривизны и имеющую величину

.

Поскольку это решающая сила и она горизонтальна, мы используем систему координат с вертикальной и горизонтальной осями. Только нормальная сила имеет горизонтальную составляющую, поэтому она должна равняться центростремительной силе, то есть

.

Поскольку автомобиль не отрывается от поверхности дороги, чистая вертикальная сила должна быть равна нулю, что означает, что вертикальные составляющие двух внешних сил должны быть равны по величине и противоположны по направлению.Из (Рисунок) мы видим, что вертикальная составляющая нормальной силы равна

.

, а единственная другая вертикальная сила — это вес автомобиля. Они должны быть равными по величине; таким образом,

Теперь мы можем объединить эти два уравнения, чтобы исключить N и получить выражение для

.

, по желанию. Решение второго уравнения относительно

и подставив его в первое, получим

Взяв арктангенс, получаем

Это выражение можно понять, рассмотрев, как

зависит от v и r .Большой

получается для большого v и маленького r. То есть дороги должны иметь крутой уклон для высоких скоростей и крутых поворотов. Трение помогает, потому что оно позволяет вам двигаться по кривой с большей или меньшей скоростью, чем если бы по кривой не было трения. Обратите внимание, что

не зависит от массы автомобиля.

Пример

Какова идеальная скорость для выхода на крутой крутой поворот?

Кривые на некоторых испытательных треках и гоночных трассах, таких как Международная гоночная трасса Дейтона во Флориде, имеют очень крутой уклон.Этот крен с помощью трения шин и очень стабильной конфигурации автомобиля позволяет преодолевать повороты на очень высокой скорости. Чтобы проиллюстрировать это, рассчитайте скорость, с которой кривая радиусом 100,0 м переходит в

градуса.

На

следует ездить, если на дороге нет трения.

Стратегия

Прежде всего отметим, что все члены в выражении для идеального угла кривой с наклоном, кроме скорости, известны; таким образом, нам нужно только переставить его так, чтобы скорость появлялась в левой части, а затем подставить известные величины.

Решение

Начиная с

получаем

Отметив, что

получаем

Значение

Это примерно 165 км / ч, что соответствует очень крутому и довольно крутому повороту. Трение в шинах позволяет автомобилю преодолевать поворот на значительно более высоких скоростях.

Самолеты также совершают развороты по крену. Подъемная сила, создаваемая силой воздуха, воздействующего на крыло, действует под прямым углом к ​​крылу.Когда самолет кренится, пилот получает большую подъемную силу, чем необходимо для горизонтального полета. Вертикальная составляющая подъемной силы уравновешивает вес самолета, а горизонтальная составляющая ускоряет самолет. Угол крена, показанный на (Рисунок), равен

.

. Мы анализируем силы так же, как и в случае поворота автомобиля по кривой.

Рис. 6.23 В крене горизонтальная составляющая подъемной силы неуравновешивается и ускоряет самолет.Обычный компонент подъемной силы уравновешивает вес самолета. Угол крена определяется как

. Сравните векторную диаграмму с диаграммой, показанной на (Рисунок).

Силы инерции и неинерциальные (ускоренные) рамки: сила Кориолиса

Что общего у взлета на реактивном самолете, поворота на автомобиле, езды на карусели и кругового движения тропического циклона? Каждый из них проявляет силы инерции — силы, которые кажутся просто возникающими в результате движения, потому что система отсчета наблюдателя ускоряется или вращается.При взлете на реактивном самолете большинство людей согласятся, что создается ощущение, будто вас толкают обратно в кресло, когда самолет ускоряется по взлетно-посадочной полосе. Однако физик сказал бы, что вы, , склонны оставаться неподвижными, в то время как сиденье толкает вас вперед. Еще более распространенный опыт происходит, когда вы делаете крутой поворот на своей машине, скажем, вправо ((рисунок)). Вы чувствуете, как будто вас отбрасывает (то есть форсированный ) влево относительно машины. Опять же, физик сказал бы, что вы, , движетесь по прямой (вспомните первый закон Ньютона), но автомобиль движется вправо, а не то, что вы испытываете силу слева.

Рис. 6.24 (a) Водитель автомобиля чувствует себя вынужденным влево по отношению к автомобилю при повороте направо. Это инерционная сила, возникающая в результате использования автомобиля в качестве системы отсчета. (б) В земной системе координат водитель движется по прямой, подчиняясь первому закону Ньютона, и машина движется вправо. Слева от водителя относительно Земли нет силы. Вместо этого справа от машины есть сила, заставляющая ее повернуть.

Мы можем согласовать эти точки зрения, исследуя используемые системы координат.Давайте сконцентрируемся на людях в машине. Пассажиры инстинктивно используют автомобиль в качестве ориентира, в то время как физик может использовать Землю. Физик мог бы сделать этот выбор, потому что Земля является почти инерциальной системой отсчета, в которой все силы имеют идентифицируемое физическое происхождение. В такой системе отсчета законы движения Ньютона принимают форму, данную в Законах движения Ньютона. Автомобиль представляет собой неинерциальную систему отсчета , потому что он ускоряется в сторону. Сила слева, воспринимаемая пассажирами автомобиля, — это сила инерции , не имеющая физического происхождения (она обусловлена ​​исключительно инерцией пассажира, а не какой-либо физической причиной, такой как напряжение, трение или гравитация).Автомобиль, как и водитель, действительно ускоряется вправо. Эта сила инерции называется силой инерции, потому что она не имеет физического происхождения, такого как гравитация.

Физик выберет ту систему отсчета, которая наиболее удобна для анализируемой ситуации. Для физика нетрудно включить инерционные силы и второй закон Ньютона, как обычно, если это удобнее, например, на карусели или на вращающейся планете. Неинерционные (ускоренные) системы отсчета используются, когда это полезно.При обсуждении движения космонавта в космическом корабле, движущемся со скоростью, близкой к скорости света, необходимо учитывать различные системы отсчета, как вы это поймете при изучении специальной теории относительности.

Давайте теперь мысленно прокатимся на карусели, а именно на быстро вращающейся игровой площадке ((Рисунок)). Вы берете карусель в качестве системы отсчета, потому что вы вращаетесь вместе. Вращаясь в этой неинерциальной системе отсчета, вы чувствуете инерционную силу, которая имеет тенденцию сбивать вас с толку; это часто называют центробежной силой (не путать с центростремительной силой).Центробежная сила — это широко используемый термин, но на самом деле его не существует. Вы должны держаться крепче, чтобы противодействовать своей инерции (которую люди часто называют центробежной силой). В системе отсчета Земли нет силы, пытающейся сбить вас с толку; мы подчеркиваем, что центробежная сила — это фикция. Вы должны держаться, чтобы заставить себя двигаться по кругу, потому что в противном случае вы бы пошли по прямой, прямо с карусели, в соответствии с первым законом Ньютона. Но сила, которую вы прикладываете, действует по направлению к центру круга.

Рис. 6.25 (a) Всадник на карусели чувствует себя так, как будто его сбивают с толку. Эту инерционную силу иногда ошибочно называют центробежной силой, пытаясь объяснить движение всадника во вращающейся системе отсчета. (б) В инерциальной системе отсчета и согласно законам Ньютона его уносит инерция (незатененный всадник имеет

и голов по прямой). Сила,

, необходим для создания кругового пути.

Этот инерционный эффект, уносящий вас от центра вращения, если нет центростремительной силы, вызывающей круговое движение, хорошо используется в центрифугах ((Рисунок)). Центрифуга вращает образец очень быстро, как упоминалось ранее в этой главе. Если смотреть из вращающейся системы координат, сила инерции выбрасывает частицы наружу, ускоряя их осаждение. Чем больше угловая скорость, тем больше центробежная сила. Но на самом деле происходит то, что инерция частиц переносит их по касательной к окружности, в то время как пробирка движется по круговой траектории под действием центростремительной силы.

Рис. 6.26. Центрифуги выполняют свою задачу по инерции. Частицы в жидком осадке оседают, потому что их инерция уносит их от центра вращения. Большая угловая скорость центрифуги ускоряет осаждение. В конечном итоге частицы контактируют со стенками пробирки, которые затем создают центростремительную силу, необходимую для их движения по кругу постоянного радиуса.

Давайте теперь рассмотрим, что происходит, если что-то движется во вращающейся системе отсчета.Например, что, если вы сдвинете мяч прямо от центра карусели, как показано на (Рисунок)? Мяч движется по прямой относительно Земли (при незначительном трении) и по изогнутой вправо траектории на поверхности карусели. Человек, стоящий рядом с каруселью, видит, как мяч движется прямо, а под ним вращается карусель. В системе отсчета карусели мы объясняем кажущуюся кривую вправо с помощью силы инерции, называемой силой Кориолиса , которая заставляет мяч изгибаться вправо.Сила Кориолиса может быть использована любым человеком в этой системе отсчета, чтобы объяснить, почему объекты следуют изогнутыми путями, и позволяет нам применять законы Ньютона в неинерциальных системах отсчета.

Рис. 6.27. Посмотрев вниз на вращение карусели против часовой стрелки, мы видим, что шар, скользящий прямо к краю, следует по изогнутой вправо траектории. Человек перемещает мяч к точке B, начиная с точки A. Обе точки поворачиваются в затемненные положения (A ‘и B’), показанные в то время, когда мяч следует изогнутой траектории во вращающейся рамке и прямой траектории в системе координат Земли. .

До сих пор мы считали Землю инерциальной системой отсчета, почти не беспокоясь о эффектах, возникающих из-за ее вращения. Однако такие эффекты и существуют — например, во вращении погодных систем. Большинство последствий вращения Земли качественно можно понять по аналогии с каруселью. Если смотреть сверху на Северный полюс, Земля вращается против часовой стрелки, как и карусель на (Рисунок). Как и на карусели, любое движение в северном полушарии Земли испытывает силу Кориолиса вправо.Прямо противоположное происходит в Южном полушарии; там сила слева. Поскольку угловая скорость Земли мала, силой Кориолиса обычно можно пренебречь, но для крупномасштабных движений, таких как характер ветра, она оказывает существенное влияние.

Сила Кориолиса заставляет ураганы в северном полушарии вращаться против часовой стрелки, тогда как тропические циклоны в южном полушарии вращаются по часовой стрелке. (Термины ураган, тайфун и тропический шторм являются региональными названиями циклонов, которые представляют собой штормовые системы, характеризующиеся центрами низкого давления, сильными ветрами и проливными дождями.) (Рисунок) помогает показать, как происходят эти вращения. Воздух течет в любую область низкого давления, а тропические циклоны имеют особенно низкое давление. Таким образом, ветры движутся к центру тропического циклона или погодной системы низкого давления на поверхности. В Северном полушарии эти внутренние ветры отклоняются вправо, как показано на рисунке, создавая циркуляцию против часовой стрелки на поверхности для зон низкого давления любого типа. Низкое давление у поверхности связано с поднимающимся воздухом, который также вызывает охлаждение и образование облаков, что делает картины низкого давления вполне заметными из космоса.И наоборот, циркуляция ветра вокруг зон высокого давления в Южном полушарии происходит по часовой стрелке, но она менее заметна, потому что высокое давление связано с опусканием воздуха, обеспечивающим чистое небо.

Рис. 6.28 (a) Вращение этого урагана в Северном полушарии против часовой стрелки является основным следствием силы Кориолиса. (б) Без силы Кориолиса воздух поступал бы прямо в зону низкого давления, например, в тропических циклонах. (c) Сила Кориолиса отклоняет ветер вправо, производя вращение против часовой стрелки.(d) Ветер, выходящий из зоны высокого давления, также отклоняется вправо, вызывая вращение по часовой стрелке. (e) Противоположное направление вращения создается силой Кориолиса в Южном полушарии, что приводит к тропическим циклонам. (кредит а и кредит е: модификации работы НАСА)

Вращение тропических циклонов и траектория шара на карусели также могут быть объяснены инерцией и вращением системы под ним. Когда используются неинерциальные системы отсчета, для объяснения искривленной траектории должны быть изобретены силы инерции, такие как сила Кориолиса.Физического источника этих сил инерции нет. В инерциальной системе отсчета инерция объясняет путь, и не обнаруживается сила без идентифицируемого источника. Любая точка зрения позволяет нам описывать природу, но взгляд в инерциальной системе отсчета является самым простым в том смысле, что все силы имеют истоки и объяснения.

Концептуальные вопросы

Если вы хотите уменьшить нагрузку (которая связана с центростремительной силой) на высокоскоростные шины, вы бы использовали шины большого или малого диаметра? Объяснять.

Определите центростремительную силу. Может ли сила любого типа (например, натяжение, сила тяжести, трение и т. Д.) Быть центростремительной силой? Может ли любое сочетание сил быть центростремительной силой?

[показывать-ответ q = ”fs-id1165039453744 ″] Показать решение [/ показывать-ответ]

[скрытый-ответ a = ”fs-id1165039453744 ″]

Центростремительная сила определяется как любая чистая сила, вызывающая равномерное круговое движение. Центростремительная сила — это не новый вид силы. Обозначение «центростремительная» относится к любой силе , которая заставляет что-то вращаться по кругу.Эта сила может быть напряжением, гравитацией, трением, электрическим притяжением, нормальной силой или любой другой силой. Любая их комбинация может быть источником центростремительной силы, например, центростремительная сила в верхней части траектории троса, проходящего через вертикальный круг, является результатом как напряжения, так и силы тяжести.

[/ hidden-answer]

Если центростремительная сила направлена ​​к центру, почему вы чувствуете, что вас «отбрасывает» от центра, когда машина движется по кривой? Объяснять.

Водители гоночных автомобилей обычно срезают углы, как показано ниже (Путь 2). Объясните, как это позволяет снимать кривую с максимальной скоростью.

[show-answer q = ”329939 ″] Показать решение [/ show-answer]
[hidden-answer a =” 329939 ″] Водитель, который срезает угол (на пути 2), имеет более плавную кривую с большей радиус. Это будет лучшая гоночная трасса. Если водитель слишком быстро завернет за угол, используя гоночную трассу, он все равно соскользнет с трассы; главное — поддерживать максимальное значение статического трения.Итак, водитель хочет максимально возможной скорости и максимального трения. Рассмотрим уравнение для центростремительной силы:

где v — скорость, а r — радиус кривизны. Таким образом, уменьшая кривизну (1 / r) пути, по которому движется автомобиль, мы уменьшаем силу, которую шины должны оказывать на дорогу, что означает, что теперь мы можем увеличить скорость v. вид водителя на Пути 1, мы можем рассуждать так: чем круче поворот, тем меньше радиус поворота; чем меньше диаметр поворота, тем больше требуется центростремительная сила.Если эта центростремительная сила не приложена, результатом будет занос. [/ Hidden-answer]

Во многих парках развлечений есть аттракционы с вертикальными петлями, как показано ниже. В целях безопасности автомобили прикреплены к рельсам таким образом, чтобы они не могли упасть. Если автомобиль преодолевает вершину с правильной скоростью, только сила тяжести будет обеспечивать центростремительную силу. Какая еще сила действует и в каком направлении, если:

(a) Автомобиль преодолевает вершину быстрее этой скорости?

(b) Автомобиль переезжает через вершину со скоростью ниже этой?

Что заставляет воду удаляться с одежды в центрифуге?

[показывать-ответ q = ”fs-id1165039477270 ″] Показать решение [/ показывать-ответ]

[скрытый-ответ a = ”fs-id1165039477270 ″]

Цилиндр сушилки создает центростремительную силу на одежде (включая капли воды), заставляя ее двигаться по круговой траектории.Когда капля воды попадает в одно из отверстий бочки, она перемещается по касательной к окружности.

[/ hidden-answer]

Когда фигурист образует круг, какая сила отвечает за его поворот? Используйте в своем ответе диаграмму свободного тела.

Предположим, что ребенок едет на карусели примерно на полпути между ее центром и краем. У нее есть коробка для завтрака, покоящаяся на вощеной бумаге, так что между ней и каруселью очень мало трения.По какому пути, показанному ниже, пойдет коробка с обедом, когда она отпустит? Ланч-бокс оставляет след в пыли на карусели. Эта тропа прямая, изогнута влево или вправо? Поясните свой ответ.

[показать-ответ q = ”60053 ″] Показать решение [/ раскрыть-ответ]
[скрытый-ответ a =” 60053 ″] Если нет трения, значит и центростремительной силы нет. Это означает, что ланч-бокс будет двигаться по касательной к окружности и, таким образом, следует по пути B. След пыли будет прямым.Это результат первого закона движения Ньютона. [/ Hidden-answer]

Чувствуете ли вы, что вас бросает в любую сторону, когда вы проезжаете поворот, идеально подходящий для скорости вашего автомобиля? В каком направлении на вас действует сила автокресла?

Предположим, что масса движется по круговой траектории на столе без трения, как показано ниже. В земной системе отсчета нет центробежной силы, оттягивающей массу от центра вращения, но есть сила, растягивающая нить, прикрепляющую массу к гвоздю.Используя концепции, связанные с центростремительной силой и третьим законом Ньютона, объясните, какая сила натягивает струну, указав ее физическое происхождение.

[показать-ответ q = ”965193 ″] Показать решение [/ раскрыть-ответ]
[скрытый-ответ a =” 965193 ″] Для поддержания кругового движения должна быть центростремительная сила; это обеспечивается гвоздем в центре. Третий закон Ньютона объясняет это явление. Сила воздействия — это сила струны на массу; сила реакции — это сила массы, действующая на струну.Эта сила реакции заставляет струну растягиваться. [/ Hidden-answer]

Когда сливают унитаз или слив из раковины, вода (и другой материал) начинает вращаться вокруг слива по пути вниз. Предполагая, что начального вращения нет и поток изначально направлен прямо к водостоку, объясните, что вызывает вращение и какое направление оно имеет в Северном полушарии. (Обратите внимание, что это небольшой эффект, и в большинстве туалетов вращение вызывается направленными струями воды.) Будет ли направление вращения измениться на противоположное, если вода будет вытесняться в канализацию?

Автомобиль объезжает поворот и наталкивается на кусок льда с очень низким коэффициентом кинетической фиксации.Автомобиль съезжает с дороги. Опишите путь, по которому машина съезжает с дороги.

[показывать-ответ q = ”fs-id116503

62 ″] Показать решение [/ показывать-ответ]

[скрытый-ответ a = ”fs-id116503

62 ″]

Поскольку радиальное трение с шинами обеспечивает центростремительную силу, а трение почти равно нулю, когда автомобиль сталкивается со льдом, автомобиль подчиняется первому закону Ньютона и съезжает с дороги по прямой, касательной к кривой. Распространенное заблуждение состоит в том, что автомобиль будет двигаться по извилистой дороге за пределами дороги.

[/ hidden-answer]

Во время одной поездки в парке развлечений всадники входят в большую вертикальную бочку и становятся у стены на ее горизонтальном полу. Бочка раскручивается, и пол падает. Всадники чувствуют себя так, как будто они прижаты к стене силой, похожей на силу гравитации. Это сила инерции, которую всадники воспринимают и используют для объяснения событий во вращающейся системе отсчета ствола. Объясните в инерциальной системе отсчета (Земля почти такая же), что прижимает всадников к стене, и определите все силы, действующие на них.

Два друга разговаривают. Анна говорит, что спутник на орбите находится в свободном падении, потому что спутник продолжает падать на Землю. Том говорит, что спутник на орбите не находится в свободном падении, потому что ускорение свободного падения не равно

.

. С кем вы согласны и почему?

[показывать-ответ q = ”fs-id11650336 ″] Показать решение [/ показывать-ответ]

[скрытый-ответ a = ”fs-id11650336 ″]

Анна права. Спутник свободно падает на Землю из-за силы тяжести, хотя сила тяжести на высоте спутника слабее, и g — это не

.

.Свободное падение не зависит от стоимости г ; то есть вы можете испытать свободное падение на Марсе, если спрыгнете с Олимпа (самого высокого вулкана в Солнечной системе).
[/ hidden-answer]

Невращающаяся система отсчета, помещенная в центр Солнца, очень близка к инерциальной. Почему это не совсем инерциальная система отсчета?

Проблемы

(a) Ребенок весом 22,0 кг катается на детской карусели, вращающейся со скоростью 40,0 об / мин. Какая центростремительная сила действует, если он равен 1.25 м от центра? (b) Какая центростремительная сила действует, если карусель вращается со скоростью 3,00 об / мин и находится на расстоянии 8,00 м от ее центра? (c) Сравните каждую силу с его весом.

[show-answer q = ”fs-id11650311 ″] Показать решение [/ show-answer]

[скрытый-ответ a = ”fs-id11650311 ″]

а. 483 Н; б. 17,4 Н; c. 2,24, 0,0807

[/ hidden-answer]

Вычислите центростремительную силу на конце лопасти ветряной турбины радиусом 100 м, вращающейся на 0.5 об / с. Предположим, что масса 4 кг.

Каков идеальный угол крена для пологого поворота радиусом 1,20 км на шоссе с ограничением скорости 105 км / ч (около 65 миль / ч), если все едут на пределе?

[показывать-ответ q = ”fs-id1165039344901 ″] Показать решение [/ показывать-ответ]

[скрытый-ответ a = ”fs-id1165039344901 ″]

[/ hidden-answer]

Какова идеальная скорость для прохождения кривой радиусом 100,0 м с наклоном

?

угол?

(а) Каков радиус бобслейного поворота с креном

?

и взято на отметке 30.0 м / с при условии идеального наклона? (b) Рассчитайте центростремительное ускорение. (c) Вам кажется это ускорение большим?

[показывать-ответ q = ”fs-id116503

09 ″] Показать решение [/ показывать-ответ]

[скрытый-ответ a = ”fs-id116503

09 ″]

а. 24,6 м; б.

г. 3,73 раза г
[/ hidden-answer]

Часть езды на велосипеде включает в себя наклон под правильным углом при повороте, как показано ниже. Чтобы быть стабильным, сила, действующая на землю, должна быть на линии, проходящей через центр тяжести.Сила, действующая на колесо велосипеда, может быть разделена на две перпендикулярные составляющие: трение параллельно дороге (оно должно обеспечивать центростремительную силу) и вертикальную нормальную силу (которая должна равняться весу системы). (а) Покажите, что

(как показано) связано со скоростью v и радиусом кривизны r поворота так же, как и для проезжей части с идеальным уклоном, то есть

(б) Вычислить

для модели 12.0-м / с разворот радиусом 30,0 м (как в гонке).

Если автомобиль движется по крутому повороту со скоростью ниже идеальной, необходимо трение, чтобы не допустить скольжения внутрь поворота (проблема на обледенелых горных дорогах). (a) Рассчитайте идеальную скорость, чтобы взять кривую радиусом 100,0 м с наклоном

.

. (b) Какой минимальный коэффициент трения необходим для того, чтобы напуганный водитель проехал по той же кривой на скорости 20,0 км / ч?

[show-answer q = ”fs-id1165038980331 ″] Показать решение [/ show-answer]

[скрытый-ответ a = ”fs-id1165038980331 ″]

а.16,2 м / с; б. 0,234

[/ hidden-answer]

Современные американские горки имеют вертикальные петли, подобные показанной здесь. Радиус кривизны вверху меньше, чем по бокам, так что центростремительное ускорение вниз вверху будет больше, чем ускорение силы тяжести, удерживая пассажиров плотно прижатыми к своим сиденьям. (a) Какова скорость американских горок в верхней части петли, если радиус кривизны там 15,0 м, а ускорение машины вниз равно 1.50 г ? (б) На какой высоте над вершиной петли американские горки должны стартовать из состояния покоя, если трение пренебрежимо мало? (c) Если он действительно начинается на 5,00 м выше, чем ваш ответ на вопрос (b), сколько энергии он потерял на трение? Его масса

.

Ребенок массой 40,0 кг находится в машине с американскими горками, которая движется по петле радиусом 7,00 м. В точке А скорость автомобиля составляет 10,0 м / с, а в точке B — 10,5 м / с. Предположим, что ребенок не держится и не пристегнут ремнем безопасности.(а) Какова сила автомобильного кресла, воздействующая на ребенка в точке А? (b) Какое усилие автомобильного кресла действует на ребенка в точке B? (c) Какая минимальная скорость требуется, чтобы удерживать ребенка на сиденье в точке A?

[раскрыть-ответ q = ”484990 ″] Показать решение [/ раскрыть-ответ]
[скрытый-ответ a =” 484990 ″] a. 179 Н; б. 290 Н; c. 8,3 м / с [/ hidden-answer]

В простой модели Бора основного состояния атома водорода электрон движется по круговой орбите вокруг фиксированного протона.Радиус орбиты

и скорость электрона

Масса электрона

. Какая сила действует на электрон?

Железнодорожные пути следуют по круговой кривой радиусом 500,0 м и имеют крен под углом

.

. Для поездов какой скорости предназначены эти пути?

[показывать-ответ q = ”fs-id11650332 ″] Показать решение [/ показывать-ответ]

[скрытый-ответ a = ”fs-id11650332 ″]

20.7 м / с

[/ hidden-answer]

Ускоритель частиц в ЦЕРНе имеет форму окружности 7,0 км. (а) Какое ускорение протонов

, которые вращаются вокруг ускорителя на

скорости света? (Скорость света

) (б) Какая сила действует на протоны?

Автомобиль объезжает кривую без кренов радиусом 65 м. Если коэффициент статического трения между дорогой и автомобилем равен 0.70, какова максимальная скорость, с которой автомобиль преодолевает поворот без пробуксовки?

[показывать-ответ q = ”fs-id1165039269152 ″] Показать решение [/ показывать-ответ]

[скрытый-ответ a = ”fs-id1165039269152 ″]

21 м / с

[/ hidden-answer]

Трасса с наклоном предназначена для движения со скоростью 90,0 км / ч. Радиус поворота 310 м. Какой угол наклона шоссе?

Глоссарий

наклонная кривая
поворот на дороге с уклоном, помогающий автомобилю преодолевать поворот
центростремительная сила
любая чистая сила, вызывающая равномерное круговое движение
Сила Кориолиса
Сила инерции, вызывающая кажущееся отклонение движущихся объектов при просмотре во вращающейся системе отсчета
Идеальное банковское дело
наклон кривой дороги, где угол наклона позволяет транспортному средству преодолевать поворот с определенной скоростью без помощи трения между шинами и дорогой; чистая внешняя сила на транспортном средстве равна горизонтальной центростремительной силе в отсутствие трения
инерционная сила
Сила, не имеющая физического происхождения
неинерциальная система отсчета
ускоренная система отсчета

6.3 Центростремительная сила — Университетская физика, том 1

Задачи обучения

К концу этого раздела вы сможете:

  • Объясните уравнение центростремительного ускорения
  • Примените второй закон Ньютона, чтобы получить уравнение для центростремительной силы
  • Используйте концепции кругового движения при решении задач, связанных с законами движения Ньютона

В «Движении в двух и трех измерениях» мы изучили основные концепции кругового движения.Объект, совершающий круговое движение, например один из гоночных автомобилей, показанных в начале этой главы, должен ускоряться, потому что он меняет направление своей скорости. Мы доказали, что это центрально направленное ускорение, называемое центростремительным ускорением, выражается формулой

, где v — скорость объекта, направленная по касательной к кривой в любой момент времени. Если нам известна угловая скорость ωω, то можно использовать

Угловая скорость показывает скорость, с которой объект поворачивает кривую, в рад / с.Это ускорение действует по радиусу криволинейной траектории и поэтому также называется радиальным ускорением.

Ускорение должно производиться силой. Любая сила или комбинация сил могут вызвать центростремительное или радиальное ускорение. Вот лишь несколько примеров: натяжение троса на тросовом шаре, сила притяжения Земли на Луне, трение между роликовыми коньками и полом катка, сила наклона проезжей части, действующая на автомобиль, и силы, действующие на трубу вращающейся центрифуги. . Любая чистая сила, вызывающая равномерное круговое движение, называется центростремительной силой.Направление центростремительной силы — к центру кривизны, то же самое, что и направление центростремительного ускорения. Согласно второму закону движения Ньютона, чистая сила равна массе, умноженной на ускорение: Fnet = ma.Fnet = ma. Для равномерного кругового движения ускорением является центростремительное ускорение: . a = ac.a = ac. Таким образом, величина центростремительной силы FcFc равна

Подставляя выражения для центростремительного ускорения acac (ac = v2r; ac = rω2), (ac = v2r; ac = rω2), мы получаем два выражения для центростремительной силы FcFc через массу, скорость, угловую скорость и радиус кривизны:

Fc = mv2r; Fc = mrω2.Fc = mv2r; Fc = mrω2.

6.3

Вы можете использовать любое более удобное выражение для центростремительной силы. Центростремительная сила F → cF → c всегда перпендикулярна траектории и указывает на центр кривизны, потому что a → ca → c перпендикулярна скорости и указывает на центр кривизны. Обратите внимание, что если вы решите первое выражение для r , вы получите

Это означает, что для данной массы и скорости большая центростремительная сила вызывает небольшой радиус кривизны, то есть крутой изгиб, как на рисунке 6.20.

Фигура 6.20 Сила трения обеспечивает центростремительную силу и численно равна ей. Центростремительная сила перпендикулярна скорости и вызывает равномерное круговое движение. Чем больше Fc, Fc, тем меньше радиус кривизны r и круче кривая. Вторая кривая имеет то же v , но больший FcFc дает меньшее r ′.

Пример 6,15

Какой коэффициент трения нужен автомобилям на плоской кривой?
(а) Рассчитайте центростремительную силу, действующую на 900.Автомобиль массой 0 кг, который преодолевает поворот радиусом 500,0 м со скоростью 25,00 м / с. (b) Предполагая, что кривая без кренована, найдите минимальный статический коэффициент трения между шинами и дорогой, поскольку статическое трение является причиной того, что автомобиль не скользит (рис. 6.21).

Фигура 6.21 Эта машина на ровной местности уезжает и поворачивает налево. Центростремительная сила, заставляющая автомобиль вращаться по круговой траектории, возникает из-за трения между шинами и дорогой. Требуется минимальный коэффициент трения, иначе автомобиль будет двигаться по кривой с большим радиусом и съезжать с проезжей части.

Стратегия
  1. Мы знаем, что Fc = mv2r.Fc = mv2r. Таким образом, Fc = mv2r = (900,0 кг) (25,00 м / с) 2 (500,0 м) = 1125 Н. Fc = mv2r = (900,0 кг) (25,00 м / с) 2 (500,0 м) = 1125 Н.
  2. На рис. 6.21 показаны силы, действующие на автомобиль на кривой без наклона (ровной поверхности). Трение направлено влево, предотвращая скольжение автомобиля, и поскольку это единственная горизонтальная сила, действующая на автомобиль, трение в данном случае является центростремительной силой. Мы знаем, что максимальное статическое трение (при котором шины катятся, но не проскальзывают) составляет μsN, μsN, где μsμs — статический коэффициент трения, а N — нормальная сила.Нормальная сила равна весу автомобиля на ровной поверхности, поэтому N = mg.N = mg. Таким образом, центростремительная сила в этой ситуации равна Fc≡f = ​​μsN = μsmg. Fc≡f = ​​μsN = μsmg. Теперь у нас есть связь между центростремительной силой и коэффициентом трения. Используя уравнение мы получаем Мы решаем это для μs, μs, отмечая, что масса сокращается, и получаем Подставляя известные, мкс = (25,00 м / с) 2 (500,0 м) (9,80 м / с2) = 0,13 мкс = (25,00 м / с) 2 (500,0 м) (9,80 м / с2) = 0,13. (Поскольку коэффициенты трения являются приблизительными, ответ дается только двумя цифрами.)
Значение
Коэффициент трения, показанный на рисунке 6.21 (b), намного меньше, чем обычно наблюдается между шинами и дорогой. Автомобиль по-прежнему движется по кривой, если коэффициент больше 0,13, потому что трение покоя — это сила реакции, которая может принимать значение меньше, но не больше, чем мксН.мксН. Более высокий коэффициент также позволит автомобилю преодолевать поворот на более высокой скорости, но если коэффициент трения меньше, безопасная скорость будет меньше 25 м / с.Обратите внимание, что масса отменяется, подразумевая, что в этом примере не имеет значения, насколько сильно загружена машина для прохождения поворота. Масса сокращается, потому что трение считается пропорциональным нормальной силе, которая, в свою очередь, пропорциональна массе. Если бы поверхность дороги была наклонной, нормальная сила была бы больше, как обсуждается далее.

Проверьте свое понимание 6.9

Автомобиль, движущийся со скоростью 96,8 км / ч, движется по круговой кривой радиусом 182,9 м по ровной проселочной дороге. Какой должен быть минимальный коэффициент статического трения, чтобы автомобиль не скользил?

Кривые с наклоном

Давайте теперь рассмотрим кривую с наклоном s , где наклон дороги помогает вам преодолевать кривую (рисунок 6.22). Чем больше угол θθ, тем быстрее вы сможете повернуть кривую. Например, гоночные трассы для велосипедов и автомобилей часто имеют крутые повороты. На «кривой с идеальным наклоном» угол θθ таков, что вы можете преодолевать поворот на определенной скорости без трения между шинами и дорогой. Мы получим выражение для θθ для кривой с идеальным наклоном и рассмотрим связанный с ним пример.

Фигура 6,22 Автомобиль на этом крутом повороте уезжает и поворачивает налево.

Для идеального крена чистая внешняя сила равна горизонтальной центростремительной силе в отсутствие трения. Составляющие нормальной силы Н в горизонтальном и вертикальном направлениях должны равняться центростремительной силе и массе автомобиля соответственно. В случаях, когда силы не параллельны, удобнее всего рассматривать компоненты вдоль перпендикулярных осей — в данном случае вертикального и горизонтального направлений.

На рис. 6.22 показана диаграмма свободного тела автомобиля на кривой без трения с наклоном.Если угол θθ идеален для скорости и радиуса, то чистая внешняя сила равна необходимой центростремительной силе. Единственными двумя внешними силами, действующими на автомобиль, являются его вес w → w → и нормальная сила дороги N → .N →. (Поверхность без трения может оказывать только силу, перпендикулярную поверхности, то есть нормальную силу.) Эти две силы должны складываться, чтобы получить результирующую внешнюю силу, горизонтальную по направлению к центру кривизны и имеющую величину mv2 / r.mv2 / р. Поскольку это решающая сила и она горизонтальна, мы используем систему координат с вертикальной и горизонтальной осями.Только нормальная сила имеет горизонтальную составляющую, поэтому она должна равняться центростремительной силе, то есть

.

Поскольку автомобиль не отрывается от поверхности дороги, чистая вертикальная сила должна быть равна нулю, что означает, что вертикальные составляющие двух внешних сил должны быть равны по величине и противоположны по направлению. Из рисунка 6.22 видно, что вертикальная составляющая нормальной силы равна Ncosθ, Ncosθ, а единственная другая вертикальная сила — это вес автомобиля. Они должны быть равными по величине; таким образом,

Теперь мы можем объединить эти два уравнения, чтобы исключить N и получить выражение для θθ, если хотите.Решив второе уравнение для N = mg / (cosθ) N = mg / (cosθ) и подставив его в первое, получим

mgsinθcosθ = mv2rmgtanθ = mv2rtanθ = v2rg.mgsinθcosθ = mv2rmgtanθ = mv2rtanθ = v2rg.

Взяв арктангенс, получаем

θ = tan − 1 (v2rg). θ = tan − 1 (v2rg).

6.4

Это выражение можно понять, рассмотрев, как θθ зависит от v и r . Большой θθ получается для большого v и маленького r. То есть дороги должны иметь крутой уклон для высоких скоростей и крутых поворотов.Трение помогает, потому что оно позволяет вам двигаться по кривой с большей или меньшей скоростью, чем если бы по кривой не было трения. Обратите внимание, что θθ не зависит от массы автомобиля.

Пример 6,16

Какова идеальная скорость для выхода на крутой крутой поворот?
Кривые на некоторых испытательных треках и гоночных трассах, таких как Международная гоночная трасса Дейтона во Флориде, имеют очень крутой наклон. Этот крен с помощью трения шин и очень стабильной конфигурации автомобиля позволяет преодолевать повороты на очень высокой скорости.Чтобы проиллюстрировать это, вычислите скорость, с которой следует двигаться по кривой радиусом 100,0 м с наклоном 31,0 ° 31,0 °, если дорога не имеет трения.
Стратегия
Прежде всего отметим, что все члены в выражении для идеального угла кривой с наклоном, кроме скорости, известны; таким образом, нам нужно только переставить его так, чтобы скорость появлялась в левой части, а затем подставить известные величины.
Решение
Начиная с

получаем

Учитывая, что tg31,0 ° = 0,609, tan31,0 ° = 0,609, получаем

v = (100.0 м) (9,80 м / с2) (0,609) = 24,4 м / с. V = (100,0 м) (9,80 м / с2) (0,609) = 24,4 м / с.
Значение
Это всего около 165 км / ч, что соответствует очень крутому и довольно крутому повороту. Трение в шинах позволяет автомобилю преодолевать поворот на значительно более высоких скоростях.

Самолеты также совершают развороты по крену. Подъемная сила, создаваемая силой воздуха, воздействующего на крыло, действует под прямым углом к ​​крылу. Когда самолет кренится, пилот получает большую подъемную силу, чем необходимо для горизонтального полета. Вертикальная составляющая подъемной силы уравновешивает вес самолета, а горизонтальная составляющая ускоряет самолет.Угол крена, показанный на рис. 6.23, равен θθ. Мы анализируем силы так же, как и в случае поворота автомобиля по кривой.

Фигура 6,23 В повороте крен горизонтальный компонент подъемной силы неуравновешен и ускоряет самолет. Обычный компонент подъемной силы уравновешивает вес самолета. Угол крена определяется как θθ. Сравните векторную диаграмму с диаграммой на рис. 6.22.

Силы инерции и неинерциальные (ускоренные) рамки: сила Кориолиса

Что общего у взлета на реактивном самолете, поворота на автомобиле, езды на карусели и кругового движения тропического циклона? Каждый из них проявляет силы инерции — силы, которые кажутся просто возникающими в результате движения, потому что система отсчета наблюдателя ускоряется или вращается.При взлете на реактивном самолете большинство людей согласятся, что создается ощущение, будто вас толкают обратно в кресло, когда самолет ускоряется по взлетно-посадочной полосе. Однако физик сказал бы, что вы, , склонны оставаться неподвижными, в то время как сиденье толкает вас вперед. Еще более распространенный опыт происходит, когда вы делаете крутой поворот на своей машине — скажем, вправо (рис. 6.24). Вы чувствуете, как будто вас отбрасывает (то есть форсированный ) влево относительно машины. Опять же, физик сказал бы, что вы, , едете по прямой (вспомните первый закон Ньютона), но автомобиль движется вправо, а не то, что вы испытываете силу слева.

Фигура 6,24 (a) Водитель автомобиля чувствует, что его заставляют двигаться влево по отношению к автомобилю, когда он поворачивает направо. Это инерционная сила, возникающая в результате использования автомобиля в качестве системы отсчета. (б) В земной системе координат водитель движется по прямой, подчиняясь первому закону Ньютона, и машина движется вправо. Слева от водителя относительно Земли нет силы. Вместо этого справа от машины есть сила, заставляющая ее повернуть.

Мы можем согласовать эти точки зрения, исследуя используемые системы координат.Давайте сконцентрируемся на людях в машине. Пассажиры инстинктивно используют автомобиль в качестве ориентира, в то время как физик может использовать Землю. Физик мог бы сделать этот выбор, потому что Земля является почти инерциальной системой отсчета, в которой все силы имеют идентифицируемое физическое происхождение. В такой системе отсчета законы движения Ньютона принимают форму, данную в Законах движения Ньютона. Автомобиль является неинерциальной системой отсчета, потому что он ускоряется в сторону. Сила слева, воспринимаемая пассажирами автомобиля, — это сила инерции, не имеющая физического происхождения (она возникает исключительно из-за инерции пассажира, а не из-за какой-либо физической причины, такой как напряжение, трение или гравитация).Автомобиль, как и водитель, действительно ускоряется вправо. Эта сила инерции называется силой инерции, потому что она не имеет физического происхождения, такого как гравитация.

Физик выберет ту систему отсчета, которая наиболее удобна для анализируемой ситуации. Для физика нетрудно включить инерционные силы и второй закон Ньютона, как обычно, если это удобнее, например, на карусели или на вращающейся планете. Неинерционные (ускоренные) системы отсчета используются, когда это полезно.При обсуждении движения космонавта в космическом корабле, движущемся со скоростью, близкой к скорости света, необходимо учитывать различные системы отсчета, как вы это поймете при изучении специальной теории относительности.

Давайте теперь мысленно прокатимся на карусели, а именно на быстро вращающейся карусели на игровой площадке (рис. 6.25). Вы берете карусель в качестве системы отсчета, потому что вы вращаетесь вместе. Вращаясь в этой неинерциальной системе отсчета, вы чувствуете инерционную силу, которая имеет тенденцию сбивать вас с толку; это часто называют центробежной силой (не путать с центростремительной силой).Центробежная сила — это широко используемый термин, но на самом деле его не существует. Вы должны держаться крепче, чтобы противодействовать своей инерции (которую люди часто называют центробежной силой). В системе отсчета Земли нет силы, пытающейся сбить вас с толку; мы подчеркиваем, что центробежная сила — это фикция. Вы должны держаться, чтобы заставить себя двигаться по кругу, потому что в противном случае вы бы пошли по прямой, прямо с карусели, в соответствии с первым законом Ньютона. Но сила, которую вы прикладываете, действует по направлению к центру круга.

Фигура 6,25 (а) Всадник на карусели чувствует себя так, как будто его сбивают с ног. Эту инерционную силу иногда ошибочно называют центробежной силой, пытаясь объяснить движение всадника во вращающейся системе отсчета. (б) В инерциальной системе отсчета и согласно законам Ньютона его уносит инерция (у незатененного всадника Fnet = 0Fnet = 0 и он движется по прямой линии). Сила FcentripetalFcentripetal необходима для создания кругового пути.

Этот инерционный эффект, уносящий вас от центра вращения, если нет центростремительной силы, вызывающей круговое движение, хорошо используется в центрифугах (рис. 6.26). Центрифуга вращает образец очень быстро, как упоминалось ранее в этой главе. Если смотреть из вращающейся системы координат, сила инерции выбрасывает частицы наружу, ускоряя их осаждение. Чем больше угловая скорость, тем больше центробежная сила. Но на самом деле происходит то, что инерция частиц переносит их по касательной к окружности, в то время как пробирка движется по круговой траектории под действием центростремительной силы.

Фигура 6,26 Центрифуги выполняют свою задачу по инерции. Частицы в жидком осадке оседают, потому что их инерция уносит их от центра вращения. Большая угловая скорость центрифуги ускоряет осаждение. В конечном итоге частицы контактируют со стенками пробирки, которые затем создают центростремительную силу, необходимую для их движения по кругу постоянного радиуса.

Давайте теперь рассмотрим, что происходит, если что-то движется во вращающейся системе отсчета.Например, что, если вы сдвинете мяч прямо от центра карусели, как показано на рис. 6.27? Мяч движется по прямой относительно Земли (при незначительном трении) и по изогнутой вправо траектории на поверхности карусели. Человек, стоящий рядом с каруселью, видит, как мяч движется прямо, а под ним вращается карусель. В системе отсчета карусели мы объясняем кажущуюся кривую вправо с помощью силы инерции, называемой силой Кориолиса, которая заставляет мяч изгибаться вправо.Сила Кориолиса может быть использована любым человеком в этой системе отсчета, чтобы объяснить, почему объекты следуют изогнутыми путями, и позволяет нам применять законы Ньютона в неинерциальных системах отсчета.

Фигура 6,27 Глядя на вращение карусели против часовой стрелки, мы видим, что шар, скользящий прямо к краю, следует по изогнутой вправо траектории. Человек перемещает мяч к точке B , начиная с точки A . Обе точки поворачиваются в заштрихованные положения ( A ’и B ’), показанные в то время, когда мяч следует по изогнутой траектории во вращающейся системе координат и по прямой траектории в системе координат Земли.

До сих пор мы считали Землю инерциальной системой отсчета, почти не беспокоясь о эффектах, возникающих из-за ее вращения. Тем не менее, такие эффекты действительно существуют — например, во вращении погодных систем. Большинство последствий вращения Земли качественно можно понять по аналогии с каруселью. Если смотреть сверху на Северный полюс, Земля вращается против часовой стрелки, как и карусель на рис. 6.27. Как и на карусели, любое движение в северном полушарии Земли испытывает силу Кориолиса вправо.Прямо противоположное происходит в Южном полушарии; там сила слева. Поскольку угловая скорость Земли мала, силой Кориолиса обычно можно пренебречь, но для крупномасштабных движений, таких как характер ветра, она оказывает существенное влияние.

Сила Кориолиса заставляет ураганы в северном полушарии вращаться против часовой стрелки, тогда как тропические циклоны в южном полушарии вращаются по часовой стрелке. (Термины ураган, тайфун и тропический шторм являются региональными названиями циклонов, которые представляют собой штормовые системы, характеризующиеся центрами низкого давления, сильными ветрами и проливными дождями.Рисунок 6.28 помогает показать, как происходят эти вращения. Воздух течет в любую область низкого давления, а тропические циклоны имеют особенно низкое давление. Таким образом, ветры движутся к центру тропического циклона или погодной системы низкого давления на поверхности. В Северном полушарии эти внутренние ветры отклоняются вправо, как показано на рисунке, создавая циркуляцию против часовой стрелки на поверхности для зон низкого давления любого типа. Низкое давление у поверхности связано с поднимающимся воздухом, который также вызывает охлаждение и образование облаков, что делает картины низкого давления вполне заметными из космоса.И наоборот, циркуляция ветра вокруг зон высокого давления в Южном полушарии происходит по часовой стрелке, но она менее заметна, потому что высокое давление связано с опусканием воздуха, обеспечивающим чистое небо.

Фигура 6,28 (а) Вращение этого урагана в Северном полушарии против часовой стрелки является главным следствием силы Кориолиса. (б) Без силы Кориолиса воздух поступал бы прямо в зону низкого давления, например, в тропических циклонах. (c) Сила Кориолиса отклоняет ветер вправо, производя вращение против часовой стрелки.(d) Ветер, выходящий из зоны высокого давления, также отклоняется вправо, вызывая вращение по часовой стрелке. (e) Противоположное направление вращения создается силой Кориолиса в Южном полушарии, что приводит к тропическим циклонам. (кредит а и кредит е: модификации работы НАСА)

Вращение тропических циклонов и траектория шара на карусели также могут быть объяснены инерцией и вращением системы под ним. Когда используются неинерциальные системы отсчета, для объяснения искривленной траектории должны быть изобретены силы инерции, такие как сила Кориолиса.Физического источника этих сил инерции нет. В инерциальной системе отсчета инерция объясняет путь, и не обнаруживается сила без идентифицируемого источника. Любая точка зрения позволяет нам описывать природу, но взгляд в инерциальной системе отсчета является самым простым в том смысле, что все силы имеют истоки и объяснения.

Автор: alexxlab

Добавить комментарий

Ваш адрес email не будет опубликован. Обязательные поля помечены *